اسئله جراحه عامه سنوات سابقه امتحان الامتياز

You might also like

Download as pdf or txt
Download as pdf or txt
You are on page 1of 110

Pediatrics questions

7/2018 , 4/2018

12/2017 ,7/2017, 4/2017 ,1/2017

10/2016 , 7/2016

sur gery

4560045770 ‫الدكتور يزن ابو غربيه‬


Page 1
General surgery
About tetanus all are true accepted; (7/2018)
a. It is a serious toxin mediated disease
b. It has mortality up to 50 %
c. Gram positive anaerobic organism
d. Culture is essential for diagnosis
e. causes diffuse muscle rigidity

Answer: D
Tetanus is diagnosed clinical with history of injury

The Goal of primary prevention is; (7/2018)


a. Early detection of disease
b. Minimize the complications of disease
c. Prevent disease onset
d. Rehabilitation of disease
e. Prevent progression of disease

Answer: C
Primary prevention: prevent the disease before it happened (vaccine)
Secondary prevention: Early detection of disease (mammogram)
Tertiary prevention: Prevent progression of disease (management program for DM)

Ideal criteria for screening test is all of the following except; (7/2018)
a. High specificity
b. low sensitivity
c. Safe ,rapid and easy
d. Cost effective
e. For significant suffering disease

Answer: B
Criteria for screening test:
1- high sensitivity
2- high specificity
3- cost effective
4- for disease with high incidence rate

Mortality from all causes of death per 1000 in population is called; (7/2018)
A. Life Expectancy
B. Infant mortality rate
C. Crude death rate
D. Crude death ratio
E. Total death rate

Answer: C
1- Crude death rate : the total number of deaths per year per 1,000 people
2- Perinatal mortality rates the sum of neonatal deaths and fetal deaths (stillbirths) per 1,000
births.
3- Maternal mortality ratio : the number of maternal deaths per 100,000 live births in same time
period.

4560045770 ‫الدكتور يزن ابو غربيه‬


Page 2
4- Maternal mortality rate : the number of maternal deaths per 1,000 women of reproductive age
in the population (generally defined as 15–44 years of age)
5- Infant mortality rate: the number of deaths of children less than 1 year old per 1,000 live births.
6- Child mortality rate: the number of deaths of children less than 5 years old per 1,000 live births
7- Standardized mortality ratio (SMR) : a proportional comparison to the numbers of deaths that
would have been expected if the population had been of a standard composition in terms of
age, gender, etc
8- Age-specific mortality rate (ASMR) : the total number of deaths per year per 1,000 people of a
given age (e.g. age 62 last birthday)
9- Cause-specific mortality rate : the mortality rate for a specified cause of death
10- Cumulative death rate: a measure of the (growing) proportion of a group that die over a
specified period (often as estimated by techniques that account for missing data by statistical
censoring)
11- Case fatality rate (CFR) : the proportion of cases of a particular medical condition that lead to
death
12- Sex-specific mortality rate : Total number of deaths in a population of a specific sex within a
given time interval

An outbreak or excessive rate of disease that rapidly spread over a wide area, crossing
international boundaries is called; (7/2018)
a. Epidemia
b. Pandemia
c. Endemia
d. Out break
e. Virulence

Answer: B
- epidemia : s a disease that spreads rapidly among many people in a community at the same
time
- endemia : disease regularly found and very common among a particular group or in
a particular area
- outbreak : a sudden rise in the incidence of a disease
- virulence : is a pathogen's or microbe's ability to infect or damage a host

One of the following infections spread mainly by airborne route; (7/2018)


a. Impetigo
b. Warts
c. Tuberculosis
d. Chicken pox
e. Malaria

Answer: C
- impetigo : contact
- warts : contact
- Chicken pox : droplets
- malaria : mosquito bit

The most common blood transfusion reaction is: (4/2018)


a. electrolyte error
b. ABO incompatibility
c. Delayed hemolysis
d. Febrile reaction
e. Transfusion related acute lung injury

4560045770 ‫الدكتور يزن ابو غربيه‬


Page 3
Answer: D
- Due to presence of recipient antibodies against some components of donors
WBCs
- Patient develops chills ,fever, N&V & headache
- Stop the transfusion + give aspirin or paracetamol

Sledging in the gallbladder and cholestatic jaundice is characteristic for while of the following
antibiotics: (4/2018)
a. Quinolones
b. Metronidazole
c. Ceftriaxone
d. Erythromycin
e. Ampicillin

Answer: C
Ceftriaxone: impress the liver function and elevate liver enzyme
Quinolones (ciprofloxacin): tendonitis
Erythromycin: diarrhea, nausea, abdominal pain, and vomiting
Ampicillin: allergic reaction

The composition of ringer lactate is: (4/2018)


a. Na 145 , Cl 109 , K4 , Ca 2.7 , Hco3 28
b. Na 130 , Cl 103 , K4 , Ca 2.7 , Hco3 28
c. Na 145 , Cl 109 , K4 , Ca 2.7 , Hco3 28
d. Na 154 , Cl 109 , K4 , Ca 4 , Hco3 28
e. Na 130 , Cl 109 , K4 , Ca 2.7 , Hco3 28

Answer: E
All IV fluid has to be isotonic positive ion = negative ion

Maintenance fluid rate to a 50kg patent; (4/2018)


a. 120 ml/Hr
b. 90 ml /Hr
c. 60 ml / Hr
d. 30 ml / Hr
e. 150 ml / Hr

Answer: B
By 4/2/1 formula
10*4 = 40
10*2 = 20
30*1 = 30
---------------
Total 90 ml/hr

Ototoxicity is characteristic of which of the following chemotherapy; (4/2018)


a. Bleomycin
b. Cisplatin
c. Cyclophosphamide
d. Vincrestin
e. Adriamycin

Answer: B

4560045770 ‫الدكتور يزن ابو غربيه‬


Page 4
Look for chemo-man

The initial treatment of choice in a patient with hyperkalemia and arrhythmia is;
a. Insulin and glucose (4/2018)
b. Amiodarone
c. Calcium resonium
d. Calcium gluconate
e. dialysis

Answer: D
Calcium gluconate to protect heart from arrhythmia

Regarding the Pre- operative management, all sentences are true Except; (12/2017)
a. A recent myocardial infarction in the most important risk factor to preoperative myocardial
infarction.
b. Patients should stop talking their regular antihypertensive drugs on the day of surgery.
c. History and examination alone are usually sufficient in evaluating the risk of bleeding
d. Low molecular weight heparin is used in Venus thromboembolism prophylaxes.
e. Poorly controlled blood sugar in the pre-operative period increases the risk of surgical site
infection

Answer: B
Pt should take antihypertensive meds before syx with sepsis of water to not have high blood
pressure during induction of anesthesia and surgery.

When total number of deaths due to measles is presented in relation to the total cause of
measles, it is best labeled as; (12/2017)
a. Cause specific death rate.
b. Incidence rate
c. Prevalence rate
d. Case fatality rate
e. Proportional mortality

Answer: A

4560045770 ‫الدكتور يزن ابو غربيه‬


Page 5
Discussed before

Imaging in surgery, All are true Except; (12/2017)


a. In abdominal X– ray, the transverse colon can be seen in various locations in the centre of the
abdomen because it is a totally infra- peritoneal organ.
b. Gallstones are usually seen on x- ray
c. The presence of gas in the small bowel raise the suspicion of bowel obstruction
d. Ultrasound is better than CT scan for gallstones detect
e. Ultrasound can assess solid organs better than gas filled organs

Answer: B
Only 10 % of gall stones appear in x ray

Spot diagnosis to differentiate lipoma and sebaceous cysts is: (12/2017)


a. Consistency
b. Attachment to deep muscle
c. Slippery edge of the lipoma and the panctum of the sebaceous cyst.
d. Compressibility
e. Fluctuation

Answer: C
Also we can use the skin picking over lipoma but not over sebaceous cyst

Informed consent for a surgical procedure involves discussion all of the following except:
(12/2017)
a. The main treatment available
b. The individual benefits and risks associated with each treatment option.
c. Wither the operation is entirely necessary and the effects of possibly not having the procedure
at all.
d. It is a acceptable not to mention some risks that may upset your patient or his family
e. It is good idea to have such discussion in the presence of the close relative.

Answer: D
Pt has to know all possible risks about the surgery

10 cases of food poisoning had been reported in hospital. 2 out these developed mild
gastrointestinal symptoms .4 out these developed moderate dehydration but recovery. The
characteristic of the organism of food poisoning that produces the severs from of the disease.
(12/2017)
a. Infectivity
b. Pathogenicity
c. Virulence
d. Communicability
e. Resistibity

Answer: C
Infectivity: is the ability of a pathogen to establish an infection
Pathogenicity: the ability of an organism to cause disease (harm the host)
Virulence: the degree of pathology caused by the organism

The following are principles of primary health care (PHC) except: (12/2017)
a. Community participation
b. Equity

4560045770 ‫الدكتور يزن ابو غربيه‬


Page 6
c. Use of sophisticated technology
d. Integrated comprehensive services.
e. Multi sectorial approach

Answer: C
PHC not using difficult technology

All sentences about screening test are true Except: (12/2017)


a. Screening test, are available for all common cancer
b. Most screening tests are not 100% sensitive.
c. No benefit has yet been shown for breast screening in women under age 40 years.
d. Prostatic specific antigen estimation is currently the best screening tool for prostatic cancer.
e. It takes approximately 10 years for colorectal polyp to develop into a cancer.

Answer: C
In women with high risk for breast cancer we should start screening before 40

All sentences about cancer surgery is true except: (12/2017)


a. It is called preventive when it removes a potential source of malignancy
b. It is called curative or primary when it is the definitive treatment for the present tumor.
c. It is called palliative when the aim is to relive symptoms and improve the quality of the life
rather than prolong survival.
d. It is called emergency when it deals with problem such as obstruction or perforation.
e. Neoadjuvant chemo or radio therapy is always required before cancer therapy.

Answer: E
Some cancer is removed surgically with no need for adjuvant therapy

A postoperative patient is being managed on a intensive care unit following subtotal colectomy
. Blood result shows the phosphate level is 0.25 mmol/L . Which one of the following
statement regarding intravenous phosphate is incorrect ? (7/2017)
A . May cause Hypercalcemia .
B . Should never be given at rate greater than 30 mmol over 6 hours .
C . Should preferentially be given via a central line .
D . May cause metastatic calcification .
E . Should be used even if the patient is asymptomatic

Answer : A
IV phosphate lead to Hypotension, hyperphophataemia, hypocalcaemia, hypernatraemia,
dehydration and metastatic calcification
potassium phosphate should given slowly to prevent arrhythmia

One of the following vaccines can't be given to immunocompromised patient ? (7/2017)


A . Hepatitis B .
B . DPT .
C . Influenza vaccine .
D . Inactivated polio vaccine .
E . Oral polio vaccine .

Answer : E
all live attenuated vaccine not given for immunocompromised patient
(MMR, varicella, yellow fever, nasal influenza, smallpox, oral rotavirus , OPV , BCG, oral
typhoid )

4560045770 ‫الدكتور يزن ابو غربيه‬


Page 7
All sentences about arteriovenous fistula are true except ? (7/2017)
A . It's complication include stenosis , aneurysm and heart failure .
B . Renal dialysis is the most common indication to create it .
C . Palpable thrill and audible bruit over the fistula indicate that is thrombosed .
D . Paleness and coldness of the hand during exercise and dialysis indicate the presence of steal
syndrome .
E . The three most common sites are radiocephalic , brachiocephalic , and brachial artery to
transposed basilic vein

Answer : C
palpable thrill and audible bruit over fistula indicate that’s the fistula is functioning

Regarding the preoperative management all sentences are true except ? (7/2017)
A . A recent myocardial infarction is the most important risk factor for perioperative
myocardial infarction .
B . Patients should take their regular Antihypertensive drug's even on the day of surgery .
C . History and examination alone are not usually sufficient in evaluating the risk of bleeding .
D . LMWH is used in venous thromboembolism prophylaxis .
E . Poorly controlled blood sugar in the preoperative period increases the risk for surgical site
infections .

Answer : C
history and examination is enough for evaluating risk of bleeding if we found something
suspicious like : ecchymoses , history of recurrent epistaxis , prolonged bleeding after trauma ,
heavy menses we have to do CBC , PT. PTT and bleeding time to look for coagulopathy

After exposure to suspected rabied animal , which of the following should be given ? (4/2017)
A . Rabies immunoglobulin and one dose of vaccine as soon as possible .
B . Rabies immunoglobulin then several doses of vaccine .
C . Rabies immunoglobulin alone .
D . Rabies immunoglobulin and antibiotic .
E . Vaccine and antibiotics .

Answer : B
- Unprovoked dog bites or bites from wild animals raise the issue of potential rabies.
- If the animal is available, it can be euthanized and the brain examined for signs of rabies.
- Otherwise, rabies prophylaxis with immunoglobulin plus vaccine is mandatory.

65 year old male has an entero_cutaneous fistula originating from the jejunum , suffered from
inflammatory bowel disease , which of the following would be the most appropriate
replacement of his losses ? (4/2017)
A . DW5%
B . N/S 3%
C . Ringer lactate
D . 0.9% sodium chloride
E . 6% sodium bicarbonate solution

Answer : C
in entero-cutaneous fistula there is loss of many electorates (HCO3 + Na) need replacement
with RL as it contain main substance

4560045770 ‫الدكتور يزن ابو غربيه‬


Page 8
Most common cause of non pitting edema ? (4/2017)
A . Iatrogenic
B . Filariasis
C . Lymph node resection surgery .
D . Leptospirosis
E . Chagas disease

Answer : C
pitting edema after pressure is applied to a small area, the indentation persists after the release
of the pressure . caused by :
1- systemic diseases
2- pregnancy in some women
3- heart failure
4- varicose veins
5- thrombophlebitis
6- insect bites
7- dermatitis.
Non-pitting edema when the indentation does not persist. caused by
1- lymphedema
2- lipedema
3- myxedema.

Which is gram negative organism ? (4/2017)


A . Group A Streptococcus .
B . MRSA .
C . Pseudomonas aeruginosa .
D . Clostridium tetani .
E . Listeria .

Gram positive
Answer : C : ABCDE + Great Number of Little Microbes Leave Many People Seriously Sick
bacteria

E: Enterococcus D: Diphtheriae C: Clostridium B: Bacillus A: Actinomyces

L: Listeria M: Mycobacterium L: Lactobacillus N: Nocardia G: Gardnerella

S: strept S: staph P: Propionibacterium M: Mycoplasma

On postoperative day 5 post hepatic resection a healthy 55 year old patient is noted to have
fever 38.6 c . Which of the following is the most common associated ? (4/2017)

A . UTI
B . Wound infection
C . Pneumonia
D . Intra_abdominal abscess
E . Intravenous catheter related infection

Answer : A

4560045770 ‫الدكتور يزن ابو غربيه‬


Page 9
Mnemonic Possible cause Diagnostic Therapy
test
POD Wind Atelectasis or Chest x ray Prevention
1-2 Postoperative followed by by:
pneumonia sputum Incentive
cultures spirometry ;
vancomycin
and
tazobactam-
pipercillin
For hospital-
acquired
Pneumonia
POD Water Urinary tract Urine Antibiotics
3-5 infection analysis appropriate
showing for the
positive organism
nitrated and
leukocyte
esterase.
urine
culture for
species and
sensitivity
POD Walking Deep vein Doppler U/S Heparin 5
5-7 thrombosis or of the days as a
thrombophlebitis extremities bridge to
of the IV access changing of Coumadin for
lines IV access 3-6 month
Must consider PE lines culture
for new onset of the IV
tachycardia and tips
chest pain
POD Wound Wound infection Physical Incision and
7 and cellulitis exam of the drainage if
wound for abscess or
erythema , fluid followed
purulent by antibiotics
discharge ,
or swelling
POD Wonder Drug fever or CT scan for CT guided
8- deep abscess examination percutaneous
15 of deep guided
fluid drainage of
collection the abscess
otherwise
surgery

The highest potassium concentrate in ? (4/2017)


A . Saliva

4560045770 ‫الدكتور يزن ابو غربيه‬


Page 10
B . Gastric juice
C . Bile
D . Ileal
E . Duodenum

Answer : A

Regarding the oxygen dissociation curve , one is true ? (4/2017)


A . Is shifted to the left by increase 2.3 _ DPG .
B . Is Sigmoid shape for both hemoglobin and myoglobin .
C . Is shifted to the left when one ascends to high altitude .
D . Is shifted to the right by an increase in the PH .
E . Is ideally 50% saturated at a PO2 of 28 mmhg

Answer : E
at PaO2 40 mmhg there will be 75% hemoglobin A saturation
at PaO2 60 mmhg there will be 90% hemoglobin A saturation
at PaO2 27 mmhg there will be 50 % hemoglobin A saturation

Oxygen –hemoglobin dissociation curve

- it is S shaped curve at PO2 60 mmHg the O2 sat is 92%


- when PO2> 60 mmHg there is small increase in O2 sat
- when PO2 < 60mmHg there is rapid decrease in O2 sat
- increase CADET shift curve to the right
- decrease CADET shift curve to the left
- CADET : CO2, Acid , 2.3-DPG ,Exercise ,Temperature
- curve shift to the right means less Hb binding to O2 and more O2 delivered to tissue
- curve shift to the left means more Hb binding to O2 and less O2 delivered to tissue

All of the following fistulas are unlikely to close spontaneously except ? (4/2017)
A . Fistulas secondary to malignancy .
B . Fistulas due to Crohn disease .
C . Fistulas with epithelized track .
D . Fistulas with long track .
E . Fistulas with foreign bodies .

Answer : D
long track fistula is easy to close spontaneously

The main reason of delay of eradication of measles in Jordan is ? (4/2017)


A . Migration or refugee from neighbor Arab countries ( Iraq , Syria ) due to unstable
conditions in their countries .
B . Immunization coverage rate is low in Jordan .
C . High cost of vaccine .
D . Cold chain of vaccine is not accurate .

Answer : A
MMR is vaccine in Jordan national vaccine program with high protective value

4560045770 ‫الدكتور يزن ابو غربيه‬


Page 11
A 50 years old man is admitted to hospital with severe bleeding and bruising . He is on
warfarin and his INR is 10 . For rapid complete reversal of his INR , which of the following
mostly used ? (4/2017)
A . Platelets .
B . FFP and vitamin K .
C . IV vitamin K .
D . Whole blood and vitamin K .
E . Fresh frozen and vitamin C .
One of the following is false regarding crystalloid solutions ?

Answer : B
discussed before

one of the follow is false regarding crysallois solution ? (4/2017)


A . Osmolarity of Ringer lactate solution 273 mosm L .
B . Ringer lactate contain 154 meq/L sodium .
C . Readily pass through semi _ permeable membrane , therefore they are considered
extravascular space expanders .
D . Have poor capillary perfusion and no allergic reaction and short intravascular half life
20_30 minutes .
E . Hartmann solution , 0.9% NaCl solution , 5% Dextrose in water are isotonic crystalloid
solutions .

Answer : B
Replacement fluid Na+ K+ Ca+ Cl- Lactate
-
Normal saline 154 154
Half normal saline 77 77
Quarter normal saline 39 39
Lactated Ringer’s 130 4 3 109 28
D5W 5% dextrose (50 g) in H2O

How much calories per liter in D_Dextrose 5% ? ( 4/2017)


A . 150
B . 170
C . 180
D . 200
E. 250

Answer : B
D5W is 5% dextrose. The concept of "%" in making up solutions is really "gram percent"
meaning "grams of .... per 100 cubic centimeters".
This means that there's 5 grams of glucose in 100 cc's of IV fluid. Therefore there are 50 grams
in a 1 liter bag.
50 grams/liter x 3.4 Calories/gram = 170 Calories per liter.

A patient has the following ABG , PH = 7.56 , PCO2 = 50 mmhg , PO2 = 85 mmhg , HCO3 =
40 meq/L . What is the acid base abnormality ? (4/2017)
A . Uncompensated metabolic alkalosis .
B . Respiratory acidosis with metabolic compensation .
C . Combined metabolic and respiratory alkalosis .
D . Metabolic alkalosis with respiratory compensation .
E . Mixed respiratory and metabolic alkalosis .

4560045770 ‫الدكتور يزن ابو غربيه‬


Page 12
Answer : D
high ph ( alkalosis ) high PCO2 ( respiratory acidosis ) high HCO3 ( metabolic alkalosis) so
there is metabolic alkalosis with respiratory compensation

Which of the following vaccines are live ? (4/2017)


A . H influenza type B
B . Diphtheria
C . OPV
D . Hepatitis B
E . H influenza A

Answer :C
Live Attenuated
Viral MMR, varicella, yellow fever, nasal Can’t be given to
influenza, smallpox, oral rotavirus , OPV pregnant or
immunocompromised
Bacterial
BCG, oral typhoid
Inactivated
Whole Virus IPV , rabies , hepatitis A Can be given to pregnant
or immunocompromised
Fractional Protein based Subunit: hepatitis B, parenteral
influenza, acellular pertussis

Polysaccharide Toxoid: diphtheria, tetanus


based Pure: pneumococcal, Hib,
meningococcal
Conjugate: Hib, pneumococcal,
meningococcal

One of the following is true regarding the electrolyte concentration in Hartmann solution ?
(4/2017)
A . Chloride 200 mmol/L .
B . Lactate 70 mmol/L .
C . Sodium 131 mmol/L .
D . Potassium 40 mmol /L .
E . Calcium 15 mmol/L .

Answer : C
discussed before

50 years old man is admitted to hospital with severe bleeding and bruising . He is on warfarin
and INR is 10 , for rapid complete reversal the best is ? (1/2017)
A. Platelets
B. FFP and vitamin K
C. IV vitamin K
D. Whole blood and vitamin K
E. FFP and vitamin C

Answer : B

4560045770 ‫الدكتور يزن ابو غربيه‬


Page 13
this pt has high INR needs antagonist for warfarin which is Vit K and needs FFP as new source
for clotting factor

A 73 years old man known of IHD presented complaining of severe abdominal pain , ABGs
shows PH = 7.3 , PCO2 = 25 mmHg , PO2 = 73 mmHg , HCO3 = 14 mmol / L , BE = _ 15.9 ,
the diagnosis is ? (1/2017)
A. Metabolic acidosis
B . Metabolic acidosis with respiratory compensation .
C . Respiratory acidosis .
D . Respiratory acidosis with metabolic compensation .
E . Metabolic and respiratory acidosis mixed

Answer : B
low Ph ( acidosis ) low PCO2 ( respiratory alkalosis ) low HCO3 ( metabolic acidosis )
so it is metabolic acidosis with respiratory compensation

A major bleeding in the thigh is best controlled in emergency room by ? (1/2017)


A . Tourniquent
B . Direct compression by hand .
C . Exploration under local anesthesia and clamp
D . Exploration and ligation under local anesthesia .
E . Send to operating room for exploration under general anesthesia
Answer : B
this is the best way to control bleeding from thigh

Which of the following is not an indication for surgical intervention in a patient with aortic
stenosis ? (1/2017)
A.Syncope
B. Angina pectoris
C.Poor ventricular function
D. Valve area less than 0.5 cm
E. Calcification of the valve

Answer : E
most old age have calcification of the aortic valve
indication of surgery in aortic valve stenosis :
8- symptomatic
9- valve crosssectionalarea is <0.75 cm2 (normal 2.5 to 3.5 cm2)
10- gradient >50 mm Hg

Most common cause of postoperative fever third day ? (1/2017)


A. atelectasis
B. UTI
C. wound infection
D. DVT
E. abscess

Answer : B
discussed before

One is true regarding the electrolyte concentration in Hartmann solution : (10/2016)


A) chloride 200 mmol / L
B) Lactate 70 mmol /L

4560045770 ‫الدكتور يزن ابو غربيه‬


Page 14
C) Na 131 mmol/L
D) K 40 mmol/L
E) Calcium 15 mmol/L

Answer : C
Discussed before

A 73 years old man, known of IHD , presented complaining of severe abdominal pain . In ABG
shows PH = 7.31 , PCO2 = 25 mmhg , PO2= 73 mmhg , HCO3 = 14 mmol /L , BE = 15.9 .
The diagnosis is : (10/2016)
A ) Metabolic acidosis
B) Metabolic acidosis with respiratory alkalosis compensation
C) Respiratory acidosis
D) Respiratory acidosis with metabolic compensation
E) Metabolic and respiratory acidosis mixed

Answer : B
low Ph ( acidosis ) low PCO2 ( respiratory alkalosis ) low HCO3 ( metabolic acidosis )

All are features of tumor lysis syndrome except : (10/2016)


A) Hyperkalemia
B) Hyperphosphatemia
C) Hypercalcemia
D) Hyperurecemia
E) High BUN

Answer : C
Tumor Lysis Syndrome
- oncologic emergency caused by massive tumor cell lysis
- most common after initiation of Cytotoxic therapy for leukemia and lymphoma
- cause hyperuricemia , hyperphosphatemia , Hyperkalemia , hypocalcemia
- uric acid precipitates in the renal tubules lead to acute kidney injury
- patient should receive prophylactic allopurinol , IV fluid with chemotherapy
- Rasburicase is alternative to allopurinal
- Alkalization of urine is controversial

50 years old man is admitted to hospital with severe bleeding and bruising . He is on warfarin
and INR is 10 , for rapid complete reversal the best is ? (10/2016)
A. Platelets
B. FFP and vitamin K
C. IV vitamin K
D. Whole blood and vitamin K
E. FFP and vitamin C

Answer : B
discussed before

4560045770 ‫الدكتور يزن ابو غربيه‬


Page 15
Influenza is one of the greatest pandemic disease because of : (10/2016)
A) Mutation of the virus
B) Spread by water
C) Long incubation period
D) High fatality rate
E) Lack of specific vaccines

Answer : A
because of the virus mutation we can be infected by influenzae every year

The most important indicator of a population health : (10/2016)


A) Crude birth rate
B) Crude mortality rate
C) Infant mortality rate
D) Maternal mortality rate
E) Total fertility rate

Answer : A
discussed before

- All of the following are true regarding Clopidogrel except : (10/2016)


A) It works by irreversibly inhibiting a receptor called P2Y12 on platelets.
B) The risk of bleeding is increased if it is used with aspirin
C) When discontinued , bleeding time normalizes in approximately 2_3 days
D) It inhibits platelet aggregation within 2 hours of oral administration
E) May cause TTP

Answer : C
- Common side effects include headache, nausea, easy bruising, itching, and heartburn . More
severe side effects include bleeding and thrombotic thrombocytopenic purpura

A 30 kg child has an estimated daily fluid requirement : (10/2016)


A) 40 ml/hour
B) 50 ml/hour
C) 60 ml/hour
D) 70 ml/hour
E) 80 ml/hour

Answer : D
rule 4/2/1
10*4 = 40
10*2= 20
10*1 = 10
totally = 70 ml /hr

The primary adverse reaction to Trastuzumab : (10/2016)


A) Liver toxicity
B) Renal toxicity
C) Thromboembolic events

4560045770 ‫الدكتور يزن ابو غربيه‬


Page 16
D) Cardiac toxicity
E) Aplastic anemia

Answer : D
side effect of trastuzumab
1- cardiomyopathy
2- pulmonary toxicity : dyspnea , ARDS ,interstitial pneumonia
3- neutropenia exacerbation
4- infusion reaction
5- diarrhea ,stomatitis
6- risk to pregnant

All of the following are true except ? (7/2016)


A . The best test for platelet function is bleeding time .
B . FFP is the most effective way to acutely correct INR .
C . Factor 8 is synthesized in the liver .
D . Calcium is required for normal coagulation

Answer : C
factor 8 and VWF synthesized in blood vessels endothelial

The principle cells involved in acute organ rejection is ? (7/2016)


A .B_cell .
B . T_cell .
C . Macrophages .
D . Platelets .
E . Monocyte .

Answer : B
Transplant rejection
Type Onset Mechanism and vessels history Type of
hypersensitivity

Host VS Hyper acute Immediat Preformed antibodies directed against the II


Graft e donor tissue . caused by accidental ABO
blood type incompatibility which is very
rare . presents while still in surgery with
thrombosis and occlusion of graft vessels
Acute Weeks to T cell mediated immune response directed IV
month against the foreign MHC . inflammation
and leukocyte infiltration of graft vessels
result . most common type
Chronic Months to T cell mediated process resulting from III and IV
years foreign MHC ( look like )self MHC
carrying and antigen . results in intimal
thickening and fibrosis of graft vessels as
well as graft atrophy
Graft VS host Varies Donor T cells in the graft proliferate and IV
attack the recipients tissue . most common
seen in bone marrow transplant . present
with diarrhea , rash and jaundice

4560045770 ‫الدكتور يزن ابو غربيه‬


Page 17
Which of the following is not an indication for surgical intervention in a patient with aortic
stenosis ? (7/2016)
A . Syncope .
B . Angina pectoris .
C . Poor ventricular function .
D . Valve area less than 0.5 cm2 .
E . Calcification of the valve

Answer : E
discussed before

A 63 year old man is restless and ataxic and has tonic spasms . Serum osmolality 330
mosm/kg . The most likely diagnosis ? (7/2016)
A . Hyponatremia
B . Hypercalcemia
C . Hyperkalemia
D . Hypernatremia
E . Hypermagnesemia

Answer : D
in Hypernatremia there is high NA + high serum osmolarity + low urine osmolarity & specific
gravity

Cryoprecipitate contains high concentrations of all of the following except ? (7/2016)


A . Fibrinogen
B . Factor 8
C . VWF
D . Fibronectin
E . Factor 11

Answer : E
Each unit (around 10 to 15 mL) of cryoprecipitate contains :

1- Fibrinogen 150–250 mg with a half-life of 100–150 hours


2- Factor VIII80–150 U with a half-life of 12 hours
3- von Willebrand factor 100-150 U with a half-life of 24 hours
4- Factor XIII 50–75 U with a half-life of 150–300 hours.
5- fibronectin; however there are no clear indications for fibronectin replacement.

Which of the following antibiotics can cause Red man syndrome if given Rapid IV ? (7/2016)
A . Clindamycin
B . Gentamicin
C . Ciprofloxacin
D . Levofloxacin
E . Vancomycin
Answer : E

4560045770 ‫الدكتور يزن ابو غربيه‬


Page 18
- is characterized by flushing and/or an erythematous rash that affects the face, neck, and upper
torso.
- These findings are due to interaction of vancomycin with MRGPRX2, a GPCR mediating IgE-
independent mast cell degranulation.

The body response to hypothermia includes all of the following except ? (7/2016)
A . Decreased platelet function
B . Decreased cardiac output
C . Tachycardia
D . Cardiac arrhythmia
E . Increased risk of wound infection

Answer : C
The EKG can show a wide variety of serious arrhythmias, including ventricular
fibrillation or ventricular tachycardia. The most characteristic finding is an elevation of the J-
point, known as Osborne waves. J-wave elevation may mimic ST-segment elevation

All of the following are true about TTP except ? (7/2016)


A . It does not lead to Hemolysis .
B . The Coombs test is negative .
C . Lower limbs petechiae are the most common presentation .
D . Seizures and coma are common complications .
E . The first line of treatment is plasma exchange

Answer : E
the first line in treatment of TTP is steroid not plasma exchange

Peripheral neuropathy with prolonged use is most commonly associated with ? (7/2016)
A . Clindamycin
B . Metronidazole
C . Ciprofloxacin
D . Gentamicin
E . Vancomycin

Answer : B
side effect of metronidazole :
1- epigastric pain
2- seizure
3- metallic taste
4- darkening urine
5- peripheral neuropathy
6- pancreatitis
7- hepatitis
8- fever
9- reversible neutropenia

The definitive management of Hyperkalemia is ? (7/2016)


A. Calcium chloride

4560045770 ‫الدكتور يزن ابو غربيه‬


Page 19
B . Dextrose
C . Dialysis
D . Fluid hydration
E. Salbutamol

Answer : C
Definitive therapy is hemodialysis in patients with renal failure or when pharmacologic therapy
is not sufficient.

One of the following is most likely to be present in a patient with severe magnesium deficiency
? (7/2016)
A .Respiratory depression
B .Cardiac depression
C .Tetany
D .Hypotension
E .Loss of patellar reflex

Answer : B
- cardiac depression (Torsades de pointes ) is the most common presentation for
hypomagnesemia
- tetany is the earliest manifestation of hypoMg

Stomach and duodenum


Life threatening causes of abdominal pain include all of the following except; (7/2018)
a. Perforation
b. Gastroduodenitis
c. Acute pancreatitis
d. Ruptured ectopic pregnancy
e. Referred pain of aortic dissection

Answer: b
All the other choices with high mortality rate except Gastroduodenitis

Helicobacter pylori may cause the following condition in the stomach and duodenum except;
(7/2018)
a. Gastric ulcer
b. Duodenal ulcer
c. Gastric lymphoma
d. Gastric adenocarcinoma
e. Gastrointestinal stromal tumor in the stomach

Answer: E
All other choices is mucosal condition except GST is submucosal and H.pylori effect the
mucosa

A 41 years old male smoker is diagnosed with low grade gastric lymphoma, after a successful
treatment he underwent surveillance upper endoscopy that showed a complete cure what is the
type of treatment he received; (7/2018)
a. Medical treatment

4560045770 ‫الدكتور يزن ابو غربيه‬


Page 20
b. Surgical treatment
c. Chemotherapy
d. Observation
e. Surgical and chemotherapy

Answer: A
Treatment for low grade gastric lymphoma is Antibiotics for 8 weeks as its cause H.pylori

Signs of metastatic Gastric carcinoma include all of the following except; (7/2018)
a. Virchow’s node
b. Blumer shelf
c. Iron- deficiency anemia
d. Sister Mary Joseph node
e. Irish's node

Answer: C
All other choice is sign of metastasis to gastric CA
1. Virchow node : metastasis to Supraclavicular lymph nodes
2. blumer shelf : metastasis to the pouch of Douglas
3. sister Mary Joseph : metastasis to umbilical lymph node
4. Irish's node : metastasis to axillary lymph node

Intrinsic factor of the stomach binds to: (4/2018)


a. Fe
b. Cu
c. B12 vitamin
d. Folate
e. K vitamin

Answer: C
Intrinsic factor binds Vit B12 until it reaches terminal ileum for absorption

Barrett's esophagus refers to: (4/2018)


a. The change in esophageal mucosa from columnar to squamous
b. The change in esophageal mucosa to transitional epithelium
c. Keratinization of the esophageal mucosa
d. Change in esophageal mucosa from squamous to columnar epithelium
e. Cancer of the esophagus

Answer: D
Barrett’s esophagus is long standing complication of GERD where the ACID change the
epithelium

A50 years old man present with epigastric pain unrelieved by proton pump inhibitors. You
perform an upper endoscopy, mass is in the mucosa only. You biopsy it is it was lympho
proliferative tissue the next appropriate therapy: (4/2018)
a. Total gasterectomy
b. Partial gasterectomy
c. Chemo therapy
d. Radio therapy
e. Triple therapy of H. pylori

Answer: E

4560045770 ‫الدكتور يزن ابو غربيه‬


Page 21
Low grade lymphoma treated with triple therapy

The right gastric artery is normally a branch off the; (4/2018)


a. Gastroduedenal artery
b. Right gastro- epiploic artery
c. Pancreato- duodenal artery
d. Proper common hepatic artery
e. Right hepatic artery

Answer: D
- left gastric arise from celiac artery
- short gastric arise from splenic artery
- left gastroepiploic arise from splenic artery
- right gastroepiploic arise gastroduodenal artery

80 year old woman presents with hematemesis . She has had intermittent epigastric pain after
food for the past 6 months . She was recently started on Diclofenac . The most likely cause for
her condition ? (4/2017)
A . Mallory Weiss tear .
B . Esophagitis .
C . Gastric malignancy .
D . Aortoenteric fistula .
E . Gastric ulcer .

Answer : E
scince pt has abdominal pain with history of using NSAID most likely ulcer

52 year old male patient was found to have Gastrinoma , which of the following is the most
likely location for Gastrinoma ? (1/2017)
A . Fundus of stomach
B . Antrum of stomach
C . The triangle formed by the junction of the second and third portion of the duodenum , the
junction of the neck and body of the pancreas and the junction of the cystic and common bile
duct .
D . The tail of pancreas
E . The triangle formed by the inferior edge of the liver , the cystic duct , and the common
hepatic duct .

Answer : C
this is the Gastrinoma triangle

Which intestinal cells have been implicated in the formation of gastrointestinal stromal tumors
? (1/2017)
A. Goblet cells
B. APUD cells
C. Cajal cells
D. Paneth cells
E. G cells

Answer : C

4560045770 ‫الدكتور يزن ابو غربيه‬


Page 22
Cajal cells s a type of interstitial cell found in the gastrointestinal tract serve
as electrical pacemakers and generate spontaneous electrical slow waves in the gastrointestinal
(GI) tract that are the basis for peristalsis and segmentation

Which of the following describes the association between sister Mary Joseph nodule and
stomach cancer ? (1/2017)
A. Metastasis left axially lymph node .
B . Metastatic left supraclavicular lymph node
C . Ovarian mass
D . Umbilical metastasis
E . Anterior nodule palpable on rectal exam

Answer : D
- Virchow sign : metastasis to supraclavicular lymph node
- Sister joseph : metastasis to umbilical node
- Blumer shelf: metastasis to pouch of Douglas

A 25 years old . Underwent an antrectomy. and vagotomy , he is complaining of nausea,


vomiting , diarrhea ,dizziness , fatigue , bloating . Within 15_30 min from meal ingestion , and
low blood sugar within 2 hours from meal . The most likely diagnosis : (10/2016)
A) Anemia
B) Jejunogastric intussusception
C) Dumping syndrome
D) Afferent loop syndrome
E) Alkaline reflex gastritis

Answer : C
Dumping Syndrome
Etiology: vagotomy and gastric resection were performed to treat severe ulcer disease.
Clinical pictures: sweating, shaking, palpitations, and lightheadedness shortly after a meal
Treatment: eat multiple small meals

Which intestinal cells have been implicated in the formation of GIST : (10/2016)
A) Goblet cells
B) APUD cells
C) Cajal cells
D) Paneth cells
E) G cells

Answer : C
discussed before

Which of the following is the most effective treatment for intractable Dumping syndrome :
(10/2016)
A) Low fat , lactose free diet
B) Serotonin antagonists
C) Low carbs , high fat diet
D) Octreotide
E) Omeprazole

Answer : D

4560045770 ‫الدكتور يزن ابو غربيه‬


Page 23
octereoide decrease gastric emptying and insulin release Which are causes of early and late
dumping respectively

Barrett esophagus ? (7/2016)


A . It is a congenital anomaly .
B . When diagnosed , should be treated by surgery .
C . Occurs in 5% of patient with Achalasia .
D . More common in females .
E . Presence of intestinal Goblet cells .

Answer : E
Barrett esophagus
Etiology:
Long-standing reflux disease the epithelium of the lower esophagus undergoes histologic
change from a normal squamous epithelium to a columnar epithelium.
Diagnosis:
Barrett esophagus >> endoscopy every 2 - 3 years
Low grade dysplasia >> endoscopy in 3 to 6 months
High grade dysplasia >> distal esophagectomy or an endoscopic mucosal resection
Treatment: PPIs
- presence of goblet cell with bad prognosis type

Which of the following is true regarding Mallory Weiss syndrome ? (7/2016)


A . Spontaneous perforation of the esophagus .
B . Forceful vomiting followed by hematemesis .
C . It's usually associated with air in the mediastinum .
D . Endoscopy is contraindicated .
E . Prompt surgical management is the best .

Answer : B
Pathogenesis: nontransmural tear of lower esophagus due to repeated retching and vomiting.
Clinical picture:
Painless upper GIT bleeding
Black stool of bleeding > 100 ml
Hematemesis
No dysphagia or odynophagia
Diagnosis: upper endoscopy
Treatment:
Resolve spontaneously
Injection of tear with epinephrine
Cauterization

The most sensitive and specific diagnostic test for Gastrinoma is ? (7/2016)
A . Octreotide scan
B . Fasting serum gastrin
C . Calcium stimulation test
D . Secretin stimulation test
E . Basal and stimulated gastric acid outputs

Answer : D
The secretin stimulation test is positive (abnormal) if there is a rise in gastrin level after the
injection of secretin.
Normally, secretin should suppress gastrin release.

4560045770 ‫الدكتور يزن ابو غربيه‬


Page 24
The treatment of choice for high grade primary gastric lymphoma ? (7/2016)
A . Total gasterectomy
B . Subtotal gasterectomy
C . Chemo_radio therapy
D . Total gastrectomy followed by chemotherapy
E . Radiation therapy

Answer : D

Which of the following describes the association between Sister Mary Joseph nodule and
stomach cancer ? (7/2016)
A . Metastatic left axiallary lymph nodes .
B . Metastatic left supraclavicular lymph node .
C . Ovarian mass .
D . Umbilical metastasis .
E . Anterior nodule palpable on rectal exam

Answer : D
Discussed before

Small Intestine
In bowel obstruction which one of the following features is suggestive of strangulation;
(7/2018?)
a. Intermittent pain
b. Acidosis
c. Alkalosis
d. Heavy watery diarrhea
e. Reducible hernia

Answer: B
In strangulation there is ischemia lead to Acidosis

Worrying features of acute abdominal pain all of the following except; (7/2018)
a. Tachycardia
b. Hypotension
c. Anorexia
d. Persistent vomiting
e. Massive rectal bleeding

Answer: C
Tachycardia and hypotension >> shock
Persistent vomiting >> sever illness lead to dehydration
Massive rectal bleeding >> ischemia

An 11 months old infant with abdominal pain is suspect of having an intussusception one is
true; (7/2018)
a. This condition occurs in 3 months old in 29% of causes
b. Ileocolic intussusceptions is most common
c. Vomiting occur lately in the presentation

4560045770 ‫الدكتور يزن ابو غربيه‬


Page 25
d. Fever excludes intussusceptions as the diagnosis
e. Plain abdominal x-ray is diagnostic

Answer: B
Intussusception occur 80 % < 2 years with fever and vomiting occur early and bile stained
For diagnosis do U/ S then air enema (safer than barium)

A thin man with chronic symptoms of proximal bowel obstruction, which vessel is the most
suggestive cause of obstruction. (7/2018)
a. Inferior vena cave
b. Superior mesenteric artery
c. Superior epigastric artery
d. Celiac trunk
e. Abdominal aorta

Answer: B
Superior mesenteric artery syndrome: duodenum is compressed between two arteries (the
aorta and the superior mesenteric artery)
- due to loss of the mesenteric fat pad (fatty tissue that surrounds the superior mesenteric artery)
in patient with significant weight loss or corrective spinal surgery for scoliosis

Note: Nutcracker syndrome is a vascular compression disorder and refers to the compression
of the left renal vein between the superior mesenteric artery (SMA) and aorta. This can lead to
renal venous hypertension, resulting in rupture of thin-walled veins into the collecting system
with resultant haematuria.

Ileal resection can result in all of the following except; (4/2018)


a. Decreased B12 and folate uptake
b. Decreased Bile salt uptake
c. Decreased iron uptake
d. Diarrhea, steatorrhea
e. Gallstones formation

Answer: C
As iron absorption done in duodenum

The most common metastasis to the small bowel is: (4/2018)


a. Lung cancer
b. Colon cancer
c. Prostate cancer
d. melanoma
e. Brain cancer

Answer: D
But the most common site of melanoma metastasis is liver

A patient with a liver metastasis related to a small bowel carcinoid tumor develop flushing and
diarrhea these symptoms are most likely caused by: (4/2018)
a. Epinephrine
b. Serotonin
c. Bradykinin
d. ACTIH
e. Gastrin

4560045770 ‫الدكتور يزن ابو غربيه‬


Page 26
Answer: B
carcinoid is tumor secrets serotonin lead to flushing, diarrhea, sweating, wheezing, right sided
heart disease, Vit B3 deficiency ( 3D : diarrhea + dermatitis + dementia )

If the anal fissure was located laterally what disease should be ruled out: (4/2018?)
a. Crohns disease
b. Ulcerative colitis
c. Gardners disease
d. FAP
e. Anal cancer

Answer: A
Crohns disease granuloma affect the whole GIT from mouth to anus can lead to anal fissure
while Ulcerative colitis affects the colon mainly

A 30 year old female came to your clinic requesting your opinion about surgery as a treatment
for her inflammatory bowel disease. Which sentence will you tell her; (12/2017)
a. Surgery is the primary treatment for crohn's disease
b. Surgery for ulcerative colitis is curative
c. Surgery is indicated as prophylaxis for colorectal cancer in crohn's disease
d. Strictures are common indications for surgery in ulcerative colitis
e. Recurrence is rare after surgery for crohn's disease.

Answer: B
Surgery is good option in UC but not in crohn’s as there high rate of recurrence in crohn’s at
the anastomosis site

A 70 years old woman presented to the emergency department with 3 days history of
worsening abdominal pain, vomiting , constipation on examination , You find that she has
abdomen distended , generalized tenderness, high pitched bowel sound . All the following are
important differential diagnosis except; (12/2017)
a. Colorectal cancer
b. Sigmoid diverticulitis
c. Gastro enteritis
d. Sigmoid volvulus
e. Femoral hernia

Answer: C
Pt in the case has intestinal obstruction and gastroenteritis is not cause of SBO

A 63 years old male presented as emergency with crampy abdominal pain for 4 days and
associated with vomiting and abdominal distension and constipation, patient denies weight
loss , his past history was only significant for previous open appendectomy and open
cholecystectomy All of the following sentences are true except : (12/2017)
a. Abdominal x- ray (upright and supine) can be postponed for few days.
b. It is essential to obtain an upright chest x-ray
c. Adhesions are possible cause for this condition
d. Abdominal and groin examination should be carried out to rule out hernia.
e. He should be admitted for intravenous fluid hydration and electrolyte replacement

Answer: A

4560045770 ‫الدكتور يزن ابو غربيه‬


Page 27
This pt has intestinal obstruction and first image for him is abdominal X ray to see air fluid
level

53 year old female presented as an emergency with crampy abdominal pain for 4 days and
associated with vomiting , abdominal distention and constipation . Patient denies weight loss .
Her past history was only significant for previous open appendectomy and open
cholecystectomy . Abdominal examination did not reveal an Incisional hernia . All of the
following sentences are true except ? (7/2017)
A . She should be admitted for intravenous fluid hydration and electrolyte replacement .
B . As female patient , groin examination can be postponed for few days .
C . Adhesion are the most likely cause for her condition .
D . It is essential is to obtain an upright chest x Ray .
E . It is essential is to obtain an abdominal x Ray ( upright and supine ) .

Answer : B
in this case we have to examine the groin to check for incarcerated hernia

Which of the following statements is true concerning Meckels diverticulum ? (4/2017)


A . It's found within 2 inches of the ileocecal valve .
B . It's a remnant of the embryonic vitelline duct .
C . Ectopic colonic epithelium is found in it .
D . Diagnosis is best made by CT_ scan .
E . It's usually found on the mesentric border of the bowel .

Answer : 2
rule of 2 : 2 feet from ileocecal valve

A 35 years old man with known ulcerative colitis , which of the following is an indication for
total proctocolectomy : (10/2016)
A) Occasional attack of colic and diarrhea
B) Sclerosing Cholangitis
C) Arthritides
D) Toxic megacolon
E) IDA

Answer : D
toxic megacolon is urgent condition need intervention

The organism most common associated with acute mesenteric Lymphadenitis is ? (7/2016)
A . Campylobacter jejuni
B . E coli.
C . Enterococcus
D Yersinia enterocolitica
E . Pinworms

Answer : D
Mesenteric lymphadenitis refers to inflammation of the mesenteric lymph nodes and is
considered present if a cluster of three or more lymph nodes, each measuring 5 mm or greater,
is detected in the right lower quadrant mesentery

4560045770 ‫الدكتور يزن ابو غربيه‬


Page 28
Numerous organisms have been cultured from mesenteric lymph nodes and blood, such
as beta-hemolytic streptococcus, Staphylococcus species, Escherichia coli, Streptococcus
viridans, Yersinia species (responsible for most cases currently), Mycobacterium tuberculosis,
Giardia lamblia, and non– Salmonella typhoid. Viruses, such as coxsackieviruses (A and B),
rubeola virus, and adenovirus serotypes 1, 2, 3, 5, and 7, have also been implicated.

All are true about malignant tumors of the small bowel except ? (7/2016)
A . Adenocarcinoma most commonly in the duodenum .
B . Adenocarcinoma and Carcinoid are the most common types .
C . Gastrointestinal stromal tumors are associated with acquired immunodeficiency syndrome .
D . Lymphoma most commonly in the Ileum .
E . Majority of small bowel malignancy are treated surgically .

Answer : C
there is no association of GIST with immunodeficiency syndrome

The best test to confirm eradication of H pylori after treatment is ? (7/2016)


A . H pylori serology
B . Urea breath test
C . Histologic biopsy
D . Rapid urease test
E . Antral mucosal biopsy and culture

Answer : B
if urea breath test is positive we have to treat ( it called eradication test )

Colon and Anal Canal


All of the following are complications of colonic Diverticulosis except; (7/2018)
a. Diverticulitis
b. Hemorrhage
c. Fistulae
d. Cancer
e. Abscesses

Answer: D
Diverticulosis can be infected lead to Diverticulitis and abscesses >> fistulae
Or bleed

Adenocarcinoma of the colon is most common associated with; (7/2018)


a. Familial adenomatous polypus’s
b. Tuberculosis of the small bowel
c. Lymphoma
d. Prolonged use of Cytotoxic chemotherapy for breast can
e. Ulcerative colitis

Answer: A
Familial adenomatous polyposis is cancerous 100% at age 40

4560045770 ‫الدكتور يزن ابو غربيه‬


Page 29
Most common presentation of left colon adenocarcinoma: (7/2018)
a. Weight loss
b. Iron deficiency anemia
c. Bowel obstruction
d. Abdominal pain
e. Rectal bleeding

Answer: C
And the most common presentation for Right colon carcinoma is anemia

All of the following are risk factors of volvulus except; (7/2018)


a. Low fiber diet
b. Constipation
c. Pregnancy
d. Laxative abuse
e. Old age

Answer: A
It high fiber diet (not low fiber diet) leads to excessive loading to sigmoid and increase risk of
volvulus

The most common location of colon cancer is: (4/2018)


a. Right color
b. Left colon
c. Transverse colon
d. Descending colon
e. Sigmoid colon

Answer: E
While the most common metastasis sites for colorectal cancer are the liver

All sentences about appendicitis are true except: (12/2017)


a. Poses sign is elicited by extension of the right hip
b. Roving’s sign is elicited by pressing in the left iliac fossa causing pain in the right iliac fossa
c. Pain is less after laparoscopic compared with open appendectomy
d. Wound infection is less after laparoscopic compared open appendectomy
e. In female patients, open approach is preferred over the laparoscopic appendectomy

Answer: E
Laparoscopy is better in female as we can see other organ like ovary to rule out other causes
before appendectomy

A 23 years old lady complains of painful defection associated with fresh per- rectal blood.
Possible diagnosis to consider is; (12/2017)
a. Hidradenitis suppurtive
b. Dermoid cyst
c. Pilonidal sinus
d. Anal fissure
e. Pruritus ani

Answer: D
Anal fissure come with painful bleeding per rectum

4560045770 ‫الدكتور يزن ابو غربيه‬


Page 30
One of the following is false concerning gastrointestinal stoma; (12/2017)
a. Patient with colorectal abdominal perineal resection require a permanent stoma
b. Ileostomy is usually fashioned with a spout to avoid skin irritation.
c. Colostomy is usually constructed in the right iliac fossa
d. Stoma may be indicated in patients with bowel ischemia
e. Patients, with Ileostomy may need water and electrolyte replacement by intravenous rout.

Answer: C
Colostomy most commonly at left iliac fossa

A 60 years old patient is diagnosed with colorectal cancer, all sentence, are true except:
(12/2017)
a. Most cases are sporadic (not related to inheritance).
b. Cure is possible if the disease is localized.
c. Chemotherapy could be given before or after surgery.
d. Bleeding per rectum usually indicates that’s the cancer in the cecum.
e. CT scan for chest, abdomen, and pelvis is the usually imaging test for staging.

Answer: D
Not necessarily mean that bleeding is come from cecum may be from colon

40 years old male with ulcerative colitis, now the exacerbation of GI symptoms. You can
assess severity with: (12/2017)
a. Erythema nodosum
b. Anemia
c. Dose of salazopyrine
d. Ankylosing spondylitis
e. Dysplasia on colonoscopy

Answer: C
dose of sulfasalazine increase with severity of disease

A 35 years old female patient underwent incision and drainage of a perianal abscess 2 months
ago now she presented to your clinic with an intermittent pain and smelly discharge from the
site of previous surgery , what is the most common problem that you would expect and you
should check for : (12/2017)
a. Perianal fistula
b. Anal fissure
c. Incontinence
d. Complication of her hemorrhoids
e. Necrotizing fasciitis

Answer: A
Perianal fistula is a common complication for perianal abscess and come with intermittent pain
and discharge because it full the secretion then drain

One of the following is false concerning gastrointestinal stoma ? (7/2017)


A. Patient with abdomen peritoneal reaction require a permanent stoma .
B. Iliostomy is usually fashioned with a spout to avoid skin irritation .
C. Colostomy is usually constructed in the Rt iliac Fossa .
D. Patient with Iliostomy may need water and electrolytes replacement by IV .
E. Stoma may be indicated in patient with bowel ischemia

4560045770 ‫الدكتور يزن ابو غربيه‬


Page 31
Answer : C
colostomy most likely on left iliac fossa not right

Regarding simple appendicitis , all are true except ? (7/2017)


A . Periumbilical pain is visceral in nature and occurs early .
B . Right iliac fossa pain is somatic in nature and occurs later .
C . Positive rebound tenderness means peritoneal irritation .
D . CT scan is the preferred imaging test for appendicitis in females and children .
E . Meckel diverticulum may mimic the clinical picture of appendicitis .

Answer : D
US is the preferred image for children and females

All sentences about appendicitis are true except ? (7/2017)


A . Mcburney point corresponds to the location of the tip of appendix .
B . Psoas sign is elicited by extension of the right hip .
C . Laparoscopy is one of the diagnostic methods .
D . Rovsing sign is elicited by pressing in the left iliac fossa causing pain in the right iliac fossa
.
E . Alvardo score is not routinely calculated for the diagnosis of appendicitis .

Answer : E
Alvarado score is a clinical scoring system used in the diagnosis of appendicitis
Alvarado score Point
Symptoms
Abdominal pain that migrates to the right iliac fossa 1
Anorexia (loss of appetite) or ketones in the urine 1
Nausea or vomiting 1
Tenderness in the right iliac fossa 2
Signs
Rebound tenderness (Blumberg sign) 1
Fever of 37.3 °C or more 1
Laboratory
Leukocytosis > 10,000 2
Neutrophilia > 70% 1
TOTAL 10
- A score of 5 or 6 is compatible with the diagnosis of acute appendicitis.
- A score of 7 or 8 indicates a probable appendicitis
- A score of 9 or 10 indicates a very probable acute appendicitis.

- obturator sign : pain on external rotation of the flexed thigh


- psoas sign : pain on right thigh extension.
- Rovsing’s sign : eliciting pain in the right lower quadrant on palpation of the left lower
quadrant.

All of the following are true about diverticulosis coli except ? (7/2017)
A . Sigmoid colon in the commonest site .
B . Are considered false diverticulum and it mucosa is lined by normal colonic cell .
C . The affected patient may present with bleeding per rectum .
D . Surgery is the primary treatment for diverticulosis .
E . Low fiber diet is a risk factor for this condition

4560045770 ‫الدكتور يزن ابو غربيه‬


Page 32
Answer : D
surgery if there is complication

40 year old male come to your clinic asking for advice about his anal fissure . One of the
following sentences is an appropriate advice ? (7/2017)
A . Surgery is primary treatment for acute anal fissure .
B . Anal fissures are usually located in the lateral aspect of the canal .
C . Anal fissure in patients with Crohn disease usually indicate malignancy .
D . Medical treatment is successful in 20% of case's .
E . Surgical treatment may be complicated by incontinence

Answer : E
- sphincterotomy can lead to stool incontinence as it will weak the anal sphincter
- most common site of anal fissure is posterior
- anal fissure is common in Crohn’s and not indicated for malignancy
- medical treatment effective in most of pt

A 42 year old morbidly obese white woman ( BMI = 40 ) presents to your office requesting
information on screening for colon cancer , she is asymptomatic and has a colonoscopy for her
is which of the following ? (7/2017)
A . She should have received a colonoscopy between age 10 _ 12 .
B . She should have received a colonoscopy between age 20 _ 25 .
C . She should have received a colonoscopy at age 40 .
D . She should receive a colonoscopy at age 45 .
E . She should receive a colonoscopy at age 50

Answer : E
this pt has no risk factor for colon cancer
risk factor for colon cancer which indicate early colonoscopy ( before 50 )
1- High fat low fiber diet
2- Family history
3- Ulcerative colitits + crohn’s disease
4- Gardner syndrome
5- Familial polyposis

Most common site of volvulus in the colorectal region ? (4/2017)


A . Descending colon .
B . Sigmoid .
C . Transverse colon .
D . Rectum .
E . Ascending colon .

Answer : B
most common site of volvulus is sigmoid then cecum

Which of the following is not a feature of Crohn disease ? (4/2017)


A . Involvement from mouth to anus .
B . Risk of flare up is decreased with smoking .
C . Macroscopically demonstrates cobblestone .
D . Endoscopically demonstrates skip lesions .
E . Histologically demonstrates transmural granulomatous inflammation .

4560045770 ‫الدكتور يزن ابو غربيه‬


Page 33
Answer : B
the flare increase with smoking for crohn’s disease but decrease with smoking for UC

A 62 year old male patient presented complaining of bloody diarrhea and cramps , he has
recently been started on Clindamycin for septic arthritis . As he is allergic to penicillin . Which
of the following is the most likely diagnosis ? (4/2017)
A . Campylobacter jujeni diarrhea .
B . Pseudomembranous colitis .
C . Salmonella typhi diarrhea .
D . Rectal cancer .
E . Hemorrhoids .

Answer : B
Etiology :
- using antibiotics change intestinal flora lead to overgrowth C.difficile

- C.difficile produce toxins A and B which bind to intestinal receptors


most common antibiotics :
1- Clindamycin
2- Fluoroquinolones
3- Cephalosporins
Clinical pictures :

- Mild diarrhea to fulminant colitis after using antibiotics


Diagnosis :
1- ELISA : in the past
2- Nucleic acid amplification assay ( LAMP , PCR) : detect toxin A , B gene
Treatment :

- Oral Metronidazole: is the drug of choice

- IV metronidazole : Unable to use oral medication

- Oral vancomycin indication : not respond to metronidazole

All of the following are true regarding colon cancer except ? (4/2017)
A . It's the second most common cause of cancer related death in Jordan
B . A ( T1 N0 M0 stage 3 ) tumor is equivalent to Duke stage C tumor
C . A majority of colorectal cancers occur in the descending colon .
D . FAP and HNPCC are two inherited causes of colorectal cancer .
E . Patients with primary sclerosing cholangitis and ulcerative colitis have increased risk of
developing colon cancer .

Answer : C
most common site of colon CA is sigmoid

4560045770 ‫الدكتور يزن ابو غربيه‬


Page 34
5_HIAA is tested in which of these disease ? (4/2017)
A . Medullary carcinoma of the breast .
B . Papillary thyroid carcinoma .
C . Giant cell carcinoma of the pancreas .
D . Carcinoid tumor .
E . Ganglioneuromas .

Answer : D
discussed before

Which of the following pathologies is thought to have no malignant potential ? (4/2017)


A . Ulcerative colitis
B . Villous adenoma
C . FAP
D . Hyperplastic polyps
E . Crohn disease

Answer : D
Type of polyps :
1- Hyperplastic : benign
2- Inflammatory : benign
3- Hamartomatous : benign
4- Adenoma : premalignant
- Tubular: less malignant
- tubulovillous
- villous: more malignant

4560045770 ‫الدكتور يزن ابو غربيه‬


Page 35
Acute appendicitis is most commonly associated with which of the following signs ? (1/2017)
A . Temperature above 40 Celsius .
B . Frequent loose stools .
C . Anorexia , abdominal pain , RLQ tenderness .
D . WBC count > 20.000 per cu ml .
E . None of above .

Answer : C
look for Alvarado score

Which of the following statements is true about drug therapy for Crohn disease ? (1/2017)
A . Corticosteroids effective treat acute attack and to prolong remission .
B . Sulfasalazine for Acute attack involving small bowel .
C . Azathioprine and immunosuppressive drugs for remission .
D . Low dose Cyclosporine for patients with both low and high disease activity .
E . All of the above .

Answer : E

All are ture about inflammatory bowel disease except ? (1/2017)


A . Crohn disease frequently presents with abdominal mass .
B . Ulcerative colitis rarely presents with obstruction, fistulizition of GIT .
C . Ulcerative colitis always affect the rectum .
D . ASCA antibodies are positive in Crohn disease .
E . Granulomas on biopsy are presents in ulcerative colitis

Answer : E
granulomas are present with crohn’s disease

Bacteremia from which of the following is associated with colon cancer ? (1/2017)
A . Streptococcus bovis
B . Difficile
C . MRSA
D . H pylori
E . Streptococcus viridans

Answer : A
pt with bacterial endocarditis with S.bovis should have colonoscopy for colon CA

Most common site of Volvulus ? (1/2017)


A. right colon
B. left colon
C. sigmoid colon
D. cecum
E. Ilium

Answer : C
most common site of volvulus is sigmoid then cecum

All are true about Crohn disease except : (10/2016)


A) Involvement from mouth to anus
B) Stenosis more seen in ulcerative colitis
C) Macroscopically cobblestoning

4560045770 ‫الدكتور يزن ابو غربيه‬


Page 36
D) Endoscopically skip lesions
E) Histologically transmural Granulomas

Answer : B
stenosis more in crohn’s disease

All are true about colorectal except : (10/2016)


A) More than half of the cancers in the recto_sigmoid
B) More common in older people
C) More common in economically and underdeveloped world
D) Risk of synchronous tumor , diagnosis in 5%
E) Patients with family history of CRC have increased risk

Answer : C
more common in high socioeconomic status

Acute appendicitis is most commonly associated with which of the following signs : (10/2016)
A) Temperature above 40 C
B) Frequent loose stools
C) Anorexia , abdominal pain , RLQ tenderness
D) WBC count > 20.000
E) None of above

Answer : C
look for Alvarado score

Which of the following statement is true about drug therapy for Crohn disease : (10/2016)
A) Corticosteroids effective treat acute attack and to prolong remission
B) Sulfasalazine for acute attack involving small bowel
C) Azathioprine and immunosuppressive drugs for remission
D) Low dose cyclosporine for patients with both low and high disease activity
E) All of the above

Answer : E

Colon pseudo_obstruction is caused by all , except : (10/2016)


A) Systemic illness
B) Electrolyte imbalance
C) Fecal impaction
D) Opioids
E) Hypothyroidism

Answer : C
fecal impaction lead to true obstruction
all other causes lead to obstruction with patent lumen as it effect the movement of intestine

4560045770 ‫الدكتور يزن ابو غربيه‬


Page 37
A 62 years old female was recently diagnosed with an adenocarcinoma of the Sigmoid colon .
You are concerned about the possibility of metastasis. Which of the following is most
suggestive of metastatic disease : (10/2016)
A) Multiple hepatic hypoechoic lesions on abdominal ultrasound
B) Mild elevation of carcinoembryonic antigen
C) Abdominal pain
D) Severe constipation
E) Abundant blood per rectum

Answer : A
metastasis appear on US

Which of the following is the best first line option for an acute exacerbation of Crohn disease :
(10/2016)
A) Sulfasalazine
B) Mesalamine
C) Natalizumab
D) Prednisone
E) Infliximab

Answer : D
prednisone is the best treatment for acute exacerbation

Which of the following is the most common neoplasm of the appendix : (10/2016)
A) Mucinous adenocarcinoma
B) Carcinoid tumor
C) Malignant mucocele
D) Lymphoma
E) Lymphosarcoma

Answer : B

Which of the following is the most reliable in confirming the diagnosis of acute appendicitis :
(10/2016)
A) Classic history of initial periumbilical pain shifting to the right lower quadrant
B) Rebound tenderness
C) Localized tenderness at Mcburneys point
D) Psaos sign presence
E) Rovsing sign presence

Answer : A
all other choices is sign of peritoneal irritation

Which of the following are true regarding FAP : (10/2016)


A) It is autosomal recessive
B) The polyps are hamartomas

4560045770 ‫الدكتور يزن ابو غربيه‬


Page 38
C) The risk of colon cancer is 100% by age 50
D) Once a polyp is detected total proctocolectomy is recommended
E) There is no association with upper GIT malignancy

Answer : D
FAP
- is autosomal dominant
- polyps are adenomatous
- risk of colon cancer 100% at age 40
- there is association with upper GIT malignancy

The appendiceal artery is a branch of ? (7/2016)


A . Right colic artery .
B . Middle colic artery .
C . Left colic artery .
D . Ileocolic artery .
E . Hypogastric artery .

Answer : D
appendiceal artery Is a terminal branch of the ileocolic artery that descends behind the
termination of the ileum and enters the mesoappendix of the vermiform appendix. It runs near
the free margin of the mesoappendix and ends in branches which supply the appendix

The most common presentation for Appendiceal adenocarcinoma is ? (7/2016)


A . Acute appendicitis .
B . Ascites .
C . Chronic anemia .
D . Palpable abdominal mass .
E . Incidental finding .

Answer : A
- majority of patients with appendiceal adenocarcinoma presents with acute appendicitis
- other features include ascites , abdominal mass , generalized abdominal pain
- less than 20 % of cases is found incidentally at surgery at time of surgery

A 65 year old female presents with a massive lower GIT bleeding . In ER BP 80/40 , HR 120
BPM . After fluid resuscitation the blood pressure 120/80 , HR 100 . A Nasogastric tube
aspirate is negative for blood. The best next step is ? (7/2016)
A . Mesenteric Arteriography .
B . Upper GI endoscopy .
C . Exploratory Laparotomy .
D . Lower GI endoscopy .
E . Observation .

Answer : D
the pt is stable now we have to lower GI endoscopy to find the source of bleeding

4560045770 ‫الدكتور يزن ابو غربيه‬


Page 39
Which of the following is true regarding ulcerative colitis and colon cancer ? (7/2016)
A . The risk of cancer unrelated to the extent of inflammation .
B . The malignancy typically develops in a polyp .
C . The risk of cancer related to disease duration .
D . Random endoscopy and biopsies ineffective .
E . CEA level are useful Guid _ when to perform a colonoscopy

Answer : C
risk of colon cancer in UC patient increase after 8-10 years

A 65 year old man presents with a 2 day history of left quadrant abdominal pain , nausea , low
grade fever . Temperature 39 , WBC 14000 , localized moderate left lower quadrant pain . The
best next step in management ? (7/2016)
A . NG tube .
B . IV antibiotics .
C . Flexible sigmoidscopy .
D . Barium Enema
E . Gastrograffin enema

Answer : B
this patient has diverticulitis need IV antibiotics then colonoscopy after 6 weeks

In female patients with a family history of HNPCC , which of the following is recommended
beginning at age 25 . In addition to screening colonoscopy ? (7/2016)
A . Annual mammography
B . Annual CT abdomen
C . Annual endometrial biopsy
D . Annual abdominal ultrasound
E . Annual brain CT

Answer : C
screening in Lynch syndrome :
1- colonoscopy q 1-2 years starting at 20-25 or 2-5 years prior to youngest age at which colon CA
was diagnosed in the family member
2- uterine ( endometrial ) biopsy annually starting at age 30
3- transvaginal US CA-125 blood test annually starting at age 30
4- upper endoscopy starting at age 30-35 repeated every 2-3 years depend on finding
5- urinanalsysis with cytology annually starting at age 25-30 years

Diagnosis of Carcinoid syndrome is best by ? (7/2016)


A . Serum Serotonin level
B . Serum substance P levels
C . 24 hour urinary 5_HIAA
D . Serum Neurotensin levels
E . Serum Neurokinin A levels .

4560045770 ‫الدكتور يزن ابو غربيه‬


Page 40
Answer : C
5 HIAA is the metabolites of secreted serotonin from carcinoid tumor

All are true about Peutz_Jeghers syndrome except ? (7/2016)


A . Typical small bowel lesion adenomatous polyps .
B . It's autosomal dominant .
C . Small bowel obstruction is the most common presentation .
D . There is increased risk of small bowel cancer .
E . There is increased risk of breast cancer .

Answer : A
in peutz –jeghers there is hamartomas polyps not adenoma

Bacteremia from which of the following organism is associated with colon cancer ? (7/2016)
A . Streptococcus bovis
B . C difficile
C . MRSA
D . H pylori
E . Streptococcus viridans

Answer : A
Discussed before

Liver
All of the following are risk factor for hepatocellular carcinoma except; (7/2018)
a. Hepatitis B virus
b. Wilson disease
c. Alcohol
d. Oral contraceptive pill
e. alpha- 1 antitrypsin deficiency

Answer: B
Wilson disease is controversy with CA

A 25 years old woman develops Jaundice and ascites 2 weeks after birth the most likely related
to; (4/2018)
a. Retaining placenta
b. portal vein thrombosis
c. Hepatocellular carcinoma
d. Biliary cystadeno carcinoma
e. Hepatitis

Answer: B
Pregnancy is high risk for thrombosis because of changing in clotting factor

One of the following suggest portal hypertension : (10/2016)


A) Low platelet

4560045770 ‫الدكتور يزن ابو غربيه‬


Page 41
B) low ALP
C) Normal INR
D) Low globulin
E) High GGT

Answer : A
pt with portal HTN present with Pancytopenia because of hypersplenism

Management of Ascites due to liver cirrhosis includes all except : (10/2016)


A) TIPS
B) Paracentesis
C) Spironolactone
D) Furosemide
E) Sodium restriction

Answer : A
TIPS used for treatment of portal HTN

All of the following are part of the child pugh scoring system for portal HTN except :
(10/2016)
A) Hemoglobin level
B) Ascites
C) Encephalopathy
D) Serum bilirubin
E) INR

Answer : A
Clinical value 1 point 2 point 3 point
Encephalopathy Non Stage 1-2 Stage 3-4
Ascites Absent Slight Moderate
Bilirubin mg/dl <2 2-3 >3
Bilirubin in <4 4-10 10
PBC or PSC
mg /dl
Albumin g/dl >3.5 2.8-3.5 >2.8
Prothrombin < 4 sec 4-6 sec >6 sec
time
INR <1.7 1.7-2.3 >2.3
Child class A = 5–6 points; Child class B = 7–9 points; Child class C =
10–15 points.
PBC : primary biliary cirrhosis
PSC : primary sclerosing cholangitis
INR : international normalized ratio.
Patients with Child class C disease will generally not tolerate a resection; patients with Child
class B disease may tolerate a limited resection

Jaundice with absent urine urobilinogen is most likely consist with ? (7/2016)
A . Hepatitis
B . Cirrhosis

4560045770 ‫الدكتور يزن ابو غربيه‬


Page 42
C . Hemolysis
D . Biliary obstruction
E . Sepsis

Answer : D
in biliary obstruction there is no bilirubin reach the intestine and so there is no urobilinogen

Which of the following is the best method to prevent a first bleed in a patient with known
esophageal varices ? (7/2016)
A .B blockers
B . TIPS surgery
C . Sclerotherapy
D . Endoscopic ligation
E . Nitrate

Answer : A
- this question asking about the method to PREVENT a first bleed the answer is B blocker as
BB lower the portal HTN

Gallbladder
All of the following are known complications of gallbladder stones except; (7/2018)
a. Obstructive jaundice
b. Cholangitis
c. Acute pancreatitis
d. Liver cirrhosis
e. Cholecustoduedenal fistula

Answer: D
Liver cirrhosis is not complications of gall stones

All of the following are risk factors for cholesterol gallbladder stones except; (7/2018)
a. Obesity
b. Crohn's disease
c. Multiparity
d. Cirrhosis liver
e. Starvation

Answer: D
In liver cirrhosis no bile secretion already

1. The primary surgery for patients with gallstone ileus is; (4/2018)
a. Open the ileum and removing obstruction
b. Whipple surgery
c. Cholecystectomy
d. Hepatojejunostomy
e. ERCP

Answer: A
Don’t do cholecystectomy as the gallbladder will be inflamed just remove the obstruction

All of the following sentences about gallstones are true except: (12/2017)

4560045770 ‫الدكتور يزن ابو غربيه‬


Page 43
a. Obesity and bariatric surgery are risk factor for gallstones formation.
b. Cholecystectomy can be performed in pregnant women.
c. Every patient undergoing cholecystectomy should be consented about bile leak.
d. Extra – corporeal shock wave, lithotripsy is not an effective treatment for gallstone s
e. Most gallstones are seen on x-ray.

Answer: E
90% of gall stones appear in US

A 35 years old lady who's otherwise fit and healthy was admitted via emergency complaining
of upper abdominal pain associated with nausea and fever her initial blood test showed WBC=
16 Hb=12 , what is the best imaging study in this situation ; (12/2017)
a. Abdominal and pelvic CT scan
b. Endoscopic ultra sound (EUS)
c. Magnetic resonance imaging with magnetic resonance cholangio pancreatograpy ( MRCP)
d. Endoscopic retrograde cholangiopanncreatograpy ( ERCP)
e. Abdominal ultrasound (US)

Answer: E
To check for acute cholecystitis

A 40 years old female previously healthy, presented as an emergency with abdominal pain
aggravated by fatty meals, associated with yellowish discolored of her sclera, vomiting, itching,
clay- color stool and dark urine. No history of weight loss, all are true except: (12/2017)
a. The cause of dark urine is the increases conjugated bilirubin in urine.
b. The serum liver function tests, with most likely shows increase alkaline phosphatase.
c. The serum will most likely show increased level of amylase with decrease level of lipase.
d. The most likely diagnosis in this patient is gallstones and stones in the common bile duct
(CBD).
e. The patient needs admission, pain killers, and rehydration. I.V fluid, + Antibiotics and
arrangement for ERCP.

Answer: C
This pt has obstructive cholecystitis most likely with stone not cancer as any weight loss
So no relation with elevation of pancreatic enzyme

A 28 years old female , previously healthy , presented as an emergency with abdominal pain
aggravated by fatty meals , and associated with jaundice , vomiting , itching , clay color stool ,
tea color urine , no history of weight loss . All are true except ? (7/2017)
A . This patient need admission , pain killer's , rehydration , IV fluids + _ antibiotics .
B . The most likely diagnosis gallstones and stone in the common bile duct .
C . The cause of clay color stool is absence of stercobilinogen from stool .
D . The cause of tea color urine is increased conjugated bilirubin in urine .
E . The serum liver function test will most likely show decreased alkaline phosphatase .

Answer : E
LVT will show increase in alkaline phosphatase

A 25 years old lady was admitted via emergency complaining of abdominal pain and suspected
to have acute gallstone pancreatitis based on symptoms and blood tests . What is the best initial
imaging study in this situation ? (7/2017)
A . Abdominal and pelvic computerized tomography .
B . Endoscopic ultrasound .

4560045770 ‫الدكتور يزن ابو غربيه‬


Page 44
C . MRCP .
D . ERCP .
E . Abdominal ultrasound

Answer : E
fist do US then ERCP

All of the following sentences about gallstones are true except ? (7/2017)
A . Gallstones in diabetic patients are more dangerous due to silent cholecystitis .
B . Obesity and sickle cell anemia are risk factors for gallstones formation .
C . Cholecystectomy can be performed in pregnant woman .
D . Every patient undergoing cholecystectomy should be consented about bile leak .
E . Extracorporeal shock wave lithotripsy is one of the effective treatments for gallstones

Answer : E
ESWL is used in kidney stones not gall stones

One of the following is not a complication of gallstones ? (4/2017)


A . Intestinal obstruction
B . Mucocele .
C . Acute cholycystitis .
D . Liver cirrhosis .
E . Pancreatitis .

Answer : D
gall stones doest effect liver

A 45 year old female . Patient presented to E.R complaining of right hypochondriac pain and
nausea with repeated vomiting for the last few hours . She recalls eating a cheese cake earlier
that day . Examination showed right hypochondriac tenderness but otherwise soft and lax . Her
laboratory results were normal . The most appropriate next investigation ? (1/2017)
A . Erect abdominal X_Ray
B . Endoscopy
C . CT abdomen
D . Ultrasound of the abdomen
E . H1 breath test for H pylori
Answer : D
this pt has cholecystitis need initially US

The risk of gallbladder cancer is increased with all of the following except ? (1/2017)
A. Obesity
B. Porcelain gallbladder
C. Merrizi syndrome
D. Male
E. aflatoxin

Answer : D
gallbladder CA is common in female

4560045770 ‫الدكتور يزن ابو غربيه‬


Page 45
The risk of gallbladder cancer is increased with all of the following except : (10/2016)
A) Obesity
B) Porcelain gallbladder
C) Mirrizzi syndrome
D) Male
E) aflatzxoin

Answer : D
discussed before

Jaundice with absent urinary urobilinogen is most consistent with : (10/2016)


A) Hepatitis
B) Liver cirrhosis
C) Hemolysis
D) CBD stone
E) Sepsis

Answer : D
as there is obstructive jaundice no absorption of urobilinogen from intestine to blood

Which of the following is false regarding gallstones type's : (10/2016)


A) Pure cholesterol stones are the commonest
B) Brown stones usually from Hemolysis
C) Black stones usually with bacterial infection
D) Pigment stones are dark because of presence of calcium bicarbonate
E) Most cholesterol stones are radiopaque

Answer : E
cholesterol gallstones is radiolucent

Spleen
Differential diagnosis of right iliac fossa pain includes all of the following except ?(4/2017)
A . Tubo _ ovarian mass .
B . Acute pyelonephritis .
C . Crohn disease .
D . Empyema of the gallbladder .
E . Splenic vein thrombosis .

Answer : E
other known causes for abdominal pain

The spleen filters all of the following particles cells except ? (4/2017)
A . Malformed erythrocytes
B . T lymphocytes
C . Malarial parasites

4560045770 ‫الدكتور يزن ابو غربيه‬


Page 46
D . Streptococcus pneumonia
E . Platelets

Answer : C
most of blood cells is removed by spleen

The spleen filters all of the following particles cells except : (10/2016)
A) Malformed RBCs
B) T lymphocytes
C) Malarial parasites
D) Streptococcus pneumonia
E) Platelets

Answer : C
Discussed before

Regarding abdominal trauma in pediatric group , one is true : (10/2016)


A) Splenectomy is not a routine for all types of splenic injuries
B) Peritoneal lavage is contraindicated
C) Blood transfusion must be started immediately
D) Ultrasound is the diagnostic tool of choice
E) Clinical exam , stabilisation , resuscitation are not of priority

Answer : A
discussed before

A 59 years old patient presented with left upper abdominal discomfort . His physical exam
reveals left upper quadrant tenderness . A CT scan of abdomen _ spleen infarction . Most
common cause of of his splenic infarction : (10/2016)
A) Aplastic anemia
B) ITP
C) Gilbert disease
D) Atrial fibrillation
E) Idiopathic pulmonary fibrosis

Answer : D
in atrial fibrillation there is mural thrombus lead to thrombosis in multiple organs

The most common organism associated with overwhelming post splenectomy sepsis :
(10/2016)
A) Group A Streptococcus
B) Haemophilus influenzae
C) Streptococcus pneumonia
D) Meningococcus
E) Babesia microtia

4560045770 ‫الدكتور يزن ابو غربيه‬


Page 47
Answer : C
overwhelming sepsis happened from encapsulated Bactria : streptococcus pneumonia ,
heamphlius influenza , neisseria meningitides

All are true about hyposplenism except ? (7/2016)


A . Associated with Radiation , alcoholism , Fanconi anemia , Amyloidosis , HIV infection .
B . Complicated with sepsis with encapsulated bacteria .
C . Complicated by Thrombocytosis and Leucocytosis .
D . Howell Jolly bodies in RBCs are absent on blood film .
E . It can occur in the presence of enlarge spleen

Answer : D
A Howell – jolly body is cytopathological finding of basophilic nuclear remnants in circulating
erythrocytes seen in hyposlpenic patient

After splenectomy for trauma the optimal timing for administration of the pneumococcal
vaccine is postoperative day ? (7/2016)
A.1
B. 4
C.7
D . 10
E . 14

Answer : E
- Non-elective splenectomy patients should be vaccinated on or after postoperative day 14.
- Elective splenectomy patients should be vaccinated at least 14 days prior to the operation.

Pancreas
Which of the following bad prognostic factor for acute pancreatitis: (7/2018)
a. Very high amylase serum level
b. Very high amylase serum level
c. Low pa2 less than 60 mmHg
d. Pseudocyst
e. Very low LDH serum level

Answer: C
It is part of Ranson Criteria:
At admission:
1. Age in years > 55 years
2. WBC count > 16000 cells/mm3
3. Blood glucose > 11.11 mol/L (> 200 mg/dL)
4. Serum AST > 250 IU/L
5. Serum LDH > 350 IU/L
Within 48 hours:
1. Serum calcium < 2.0 mol/L (< 8.0 mg/dL)
2. Hematocrit fall > 10%

4560045770 ‫الدكتور يزن ابو غربيه‬


Page 48
3. Oxygen (hypoxemia PaO2 < 60 mmHg)
4. BUN increased by 1.8 or more mol/L (5 or more mg/dL) after IV fluid hydration
5. Base deficit (negative base excess) > 4 mEq/L
6. Sequestration of fluids > 6 L

A 56 years old woman presents to the emergency room with severe abdominal pain. she
occasionally drinks you get routine labs; amylase 24000, lipase 7000 WBC 17000 BP 90/40
HR 120, You examine her and she is fairly tender in the epigastric area the most appropriate
next step: (4/2018)
a. Start IV fluid and get a CT scan
b. Start IV fluid and get a MRI
c. Start IV fluid and get an U/S
d. Start IV fluid and go to operation
e. Start IV fluid and observation

Answer: A
To check for necrotizing pancreatitis

A 50 years old woman with a pancreatic mass in the tail of the pancreas develops most likely
diagnosis is: (4/2018)
a. Glucagnoma
b. Gastrinoma
c. Somatostationoma
d. Vipoma
e. Insulinoma

Answer: D
The patient has WDAH syndrome:
1- W: watery
2- D: diarrhea
3- H : hypoK
4- A: achlorhydria

The most common presenting symptom in patient with pancreatic cancer is: (12/2017)
a. Mid epigastric pain
b. Anorexia
c. Malaise
d. Nausea
e. Anemia

Answer: B
The most common presentation of pancreatic Ca is anorexia and weight loss
Which is not a cause of pancreatitis; (12/2017)
a. Hypercalcemia
b. Hypokalemia
c. Hypertriglycridemia
d. Obstruction ampulla of vater
e. Thiazides

Answer: B
Causes of pancreatitis: I GET SMASHED
1. I: idiopathic
2. G: gallstones

4560045770 ‫الدكتور يزن ابو غربيه‬


Page 49
3. E: ethanol (alcohol)
4. T: trauma
5. S: steroids
6. M: mumps (and other infections) / malignancy
7. A: autoimmune
8. S: scorpion stings/spider bites
9. H: hyperlipidaemia/hypercalcaemia/hyperparathyroidism (metabolic disorders)
10. E: ERCP
11. D: drugs

A 56 years old woman presented with sever upper abdominal pain related to her back and
associated with nausea and vomiting , her serum lipase level was 6300 u/ L . The patient was
given intravenous analgesia . What is the best and most important next step in management ?
(7/2017)
A . Begin intravenous fluid resuscitation and oxygen supplement .
B . Begin prophylactic antibiotics to minimize the risk of pancreatic infection .
C . Change her diet to clear liquids by mouth only .
D . Obtain a contrast enhanced CT scan of the abdomen .
E . Obtain a right upper quadrant ultrasound .

Answer : A
first line treatment in acute pancreatitis is IV fluid

All are common of neonatal obstruction except ? (4/2017)


A.Volvulus
B. Intussusception
C. Meconium ileus
D. Annular pancreas
E.,Strangulated inguinal hernia

Answer : D
annular pancreas present with polyhydramnios (an excess of amniotic fluid), low birth weight,
and feeding intolerance immediately after birth

A 39 year old man presents with severe epigastric pain and right upper quadrant pain .
Radiating to the back associated with nausea and vomiting . Inspection of the abdomen shows
bruising below the inguinal ligament . This sign is called ? ( 4/2017 )
A . Cullen sign
B . Boas sign
C . Fox sign
D . Grey turner sign
E . Murphy's sign

Answer : C
Cullen sign is superficial edema and brusing in the subcutaneous fatty tissue in acute
pancreatitis
Boas sign : increase or altered sensitivity below right scapula in acute cholecystitis
grey turner sign : bruising of the flanks in acute pancreatitis
Murphy’s sign : stop breathing when examine the gall bladder in acute cholecystitis

A 50 year old man presented to E.R complaining of severe epigastric pain of 5 hours duration .
The pain is radiating to his back associated with recurrent vomiting . His amylase is 2017 u/ L .
All of the following criteria are used to predict the severity of the case during admission except

4560045770 ‫الدكتور يزن ابو غربيه‬


Page 50
? (4/2017)
A . Age over 55 years
B . Blood sugar more than 200 mg/dl .
C . AST more than 250 u/ L
D . WBC less than 5000 /UL
E . LDH more than 350 u/ L

Answer : D
Ranson criteria

A 50 years old man , presented to E.R with severe Epigastric pain of 5 hours duration . The
pain radiating to back , associated with recurrent vomiting , Amylase level is high , all of the
following criteria are used to peridict mortality and morbidity during admission except :
(10/2016)
A) Age over 55 years
B) Blood sugar more than 200 mg / dl
C) AST more than 250 u/ L
D) WBC less than 5000/mm3
E) LDH more than 350 u/ L

Answer : D
discussed before

Most common functional pancreatic endocrine neoplasm is : (10/2016)


A) Vipoma
B) Glucagonoma
C) Somatostatinoma
D) Gastrinoma
E) Insulinoma

Answer : E
this is the most pancreatic neoplasm

A 45 year old female presents with intermittent vague right upper quadrant pain and jaundice .
In addition she notes having floating fatty stool . Ultrasound : gallstone in gallbladder and mass
in pancreatic head . The most likely diagnosis ? (7/2016)
A . Vipoma
B . Glucagonoma
C . Somatostatinoma
D . Gastrinoma
E . Insulinoma

Answer : C
Insulinoma:
- Most common tumor I n islet of langerhans
Symptoms: Whipple triad
1- low BG< 45
2- symptoms of hypoglycemia: Tachycardia + palpitation + shaking
3- Resolve symptoms with oral glucose

4560045770 ‫الدكتور يزن ابو غربيه‬


Page 51
Diagnosis:
1- High insulin /glucose ratio
2- High C peptide and proinsulin
3- CT, MRI , somatostatin receptor scintigraphy
Treatment: surgical exploration and resection

Glucagonomas:
- excess glucagon ( tumor of alpha cell )
symptoms :
1- type II diabetes,
2- hypoaminoacidemia
3- anemia, weight loss
4- skin rash, necrolytic migratory erythema.
Diagnosis :
1- biopsy of the skin rash
2- plasma glucagon levels (usually >1,000 pg/mL).
3- CT scan.
Treatment : Resection is indicated in fit
patients after nutritional support, even if metastases are present

VIPOMA : (vip : vasoctive intestinal peptide )


Symptoms : WDHA syndrome (Verner–Morrison syndrome )
1- W :watery
2- D : diarrhea
3- H : hypoK
4- A :achlorhydria
Diagnosis : high fasting vip levels (>190 pg/mL)
Treatment :
1- Octreotide
2-VIPomas occur in the distal pancreas distal pancreatectomy

Somatostatinoma : These tumors are frequently located in the head of the pancreas
Symptoms : diabetes, steatorrhea, and cholelithiasis.

Management of pancreatic lymphoma is by ? (7/2016)


A . Pancreaticoduodenectomy
B . Chemotherapy
C . Radiotherapy
D . Pancreaticoduodenectomy followed by radio
E . Chemo_radio therapy

Answer : B

Hernia
All of the following statements are true about inguinal canal except; (7/2018)
a. Anterior wall is external oblique, internal oblique muscles, and superficial inguinal ring.

4560045770 ‫الدكتور يزن ابو غربيه‬


Page 52
b. Posterior wall is transversalis fascia, conjoint Tendon , and deep inguinal ring.
c. Inguinal canal transmit the genitofemoral nerve.
d. Superior wall is lacunar ligament and external oblige muscle.
e. Inferior wall is inguinal ligament
Answer: D
Walls of inguinal canal:
- Anterior wall: aponeurosis of the external oblique, internal oblique muscle laterally.
- Posterior wall: transversalis fascia.
- Roof: transversalis fascia, internal oblique, and transversus abdominis.
- Floor: inguinal ligament (a ‘rolled up’ portion of the external oblique aponeurosis), lacunar
ligament.

Contents of inguinal canal:


1- Spermatic cord (males only): contains neurovascular and reproductive structures that supply
and drain the testes.
2- Round ligament (females only): originates from the uterine horn and travels through the
inguinal canal to attach at the labia majora.
3- Ilioinguinal nerve : contributes towards the sensory innervation of the genitalia
- Note: only travels through part of the inguinal canal, exiting via the superficial inguinal ring (it
does not pass through the deep inguinal ring)
- This is the nerve most at risk of damage during an inguinal hernia repair.
1- Genital branch of the genitofemoral nerve : supplies >>
A. cremaster muscle and anterior scrotal skin in males
B. Skin of the mons pubis and labia majora in females.
- The walls of the inguinal canal are usually collapsed around their contents, preventing other
structures from potentially entering the canal and becoming stuck.

Rings:
1- deep (internal) ring :
- Above the midpoint of the inguinal ligament.
- Lateral to the epigastric vessels.
- Is created by the transversalis fascia, which invaginates to form a covering of the contents of
the inguinal canal.
1- superficial (external) ring
- Lies just superior to the pubic tubercle.
- triangle shaped opening, formed by the evagination of the external oblique
- Forms another covering of the inguinal canal contents.
- Contains intercrural fibers, which run perpendicular to the aponeurosis of the external
oblique and prevent the ring from widening.

All of the following is differential diagnosis of Femoral hernia except; (7/2018)


a. Bartholin gland cyst
b. Lipoma
c. Cold abscess
d. Varicose node
e. Inguinal hernia

Answer: A
Bartholin gland cyst near to vulvae

Which artery is considered to be differentiate between site of origin in inguinal hernia; (7/2018)
a. Cremastric artery
b. Superior epigastric artery

4560045770 ‫الدكتور يزن ابو غربيه‬


Page 53
c. Inferior epigastric artery
d. Femoral artery
e. Superficial circumflex iliac artery

Answer: C
Inferior epigastric artery is the lateral border for hesselbach triangle. While the inguinal
ligments forms the inferior border and rectus abdominis is the medial border

All are true about femoral hernia except; (7/2018)


a. Most common groin hernia in women
b. It goes into femoral canal
c. It goes below inguinal ligament
d. It lies medial to femoral vein
e. It is high risk for strangulation

Answer: A
Most common hernia in women is indirect inguinal hernia but the femoral hernia is common in
female

The most common injured nerve with inguinal hernia repair; (4/2018)
a. Ilioinguinal hernia repair
b. Genitofemoral nerve
c. Sciatic nerve
d. Lumbosacral nerve
e. Femoral nerve

Answer: A
As this nerve pass through inguinal canal

A 65 years old woman presents with a tender medial thigh mass the pain increases with internal
rotation of the thigh the most likely Dx: (4/2018)
a. Inguinal hernia
b. Obturator hernia
c. Ventral hernia
d. Femoral hernia
e. Howship – Romberg hernia.

Answer: B
The patient has sign of Obturator nerve compression

A 2 year old is brought into your office for an umbilical hernia. All of the following are true
except; (4/2018)
a. Patient should have surgery at about age 5 year if it fails to close
b. Patient is at high risk for incarceration
c. This hernia is increased in African patients.
d. This hernia is increased in premature patients.
e. Associated with congenital Hypothyroidism

Answer: B
Most umbilical hernia resolve spontaneously and we can wait for 4-5 years of age

4560045770 ‫الدكتور يزن ابو غربيه‬


Page 54
A 50 year old male felt a pain in his groin after lifting a heavy box. He was advised on
operation during the surgery a hernial sac with a small knuckle of intestine was found
protruding just below the inguinal ligament the hernia was diagnosed as; (12/2017)
a. Congenital inguinal hernia
b. Direct inguinal hernia
c. Femoral hernia
d. Incisional hernia
e. Indirect inguinal hernia

Answer: C
Femoral hernia is below inguinal hernia

All the following sentences about hernia are true accepted; (12/2017)
a. Inguinal hernia is the most common in both females and males
b. Incisional hernia is more common after open compared with laparoscopic operation
c. Open is always better and safer than laparoscopic approach for hernia repair
d. Divarication of rectus abdominis muscles in elderly does not necssionate surgical repair
e. Morbidity is increased after emergency compared with elective repair of hernia.

Answer: C
Open hernia repair is not better than laparoscopy from cosmetic point view

The commonest hernia type that can been strangulated is; (12/2017)
a. Incisional
b. Inguinal
c. Para- umbilical
d. Femoral
e. Linea alba

Answer: D
Commonest hernia can be strangulated is femoral

All sentences about abdominal hernias are true except ? (7/2017)


A . Most umbilical hernias in adult are acquired .
B . Incisional hernia results from a failure of fascial tissue to heal and close after surgery .
C . Incisional hernia is more common after open compared with laparoscopic operation .
D . Lumbar hernia is not common hernia .
E . Divarication of rectus abdominis muscles in elderly necessities surgical repair .

Answer : E
if no bowel herniation or incarceration no need for surgery

All of the following sentences about hernia are true except ? (7/2017)
A . Inguinal hernia is the most common hernia in both females and male's .
B . Femoral hernias are most common in females than male's .
C . Morbidity is increased after emergency compared with elective repair of hernia .
D . Urinary hesitancy and straining increase the risk for hernia formation .
E . Open is always better and safer than laparoscopic approach for hernia repair .

Answer : E
in open hernia repair there is high risk of recurrence it is not always better

4560045770 ‫الدكتور يزن ابو غربيه‬


Page 55
All of the following is true regarding the femoral triangle except ? (4/2017)
A . The inguinal ligament forms the base .
B . The medial border is formed by the lateral border of the adductor longus .
C . The lateral border is formed by the medial border of the Sartorius .
D . The floor is formed by the adductor longus and pectineus muscle medially .
E . The floor is formed by the Iliopsoas muscle laterally

Answer : B
the medial border of femoral triangle formed by medial border of the adductor longus

Most common hernia in females is ? (1/2017)


A. Femoral hernia
B. Umbilical hernia
C. Obturator hernia
D. Direct inguinal hernia
E. Indirect inguinal hernia

Answer : E
indirect inguinal hernia is the most common hernia at all in male and female but femoral hernia
more common in female

One is false? (1/2017)


A . Femoral artery is lateral to femoral vein in the inguinal area .
B . Femoral nerve is present within the femoral sheath .
C . Femoral triangle is bounding by Sartorius muscle laterally .
D . Siatic nerve injury can be avoided by giving im injection on the upper lateral quadrant on
the gluteal area .
E . Great saphenous vein drain to femoral vein below the inguinal ligament .

Answer : B
femoral nerve enters the femoral triangle by passing beneath the inguinal ligament, just lateral
to the femoral artery. In the thigh, the nerve lies in a groove between iliacus muscle and psoas
major muscles, outside the femoral sheath, and lateral to the femoral artery.

Most common hernia in females : (10/2016)


A) Femoral hernia
B) Umbilical hernia
C) Obturator hernia
D) Direct inguinal hernia
E) Indirect inguinal hernia

Answer : E
Discussed before

All of the following is true regarding the femoral triangle except : (10/2016)
A) The inguinal ligament forms the base
B) The medial border _ lateral border of adductor longus muscle
C) The lateral border _medial border of sartorius
D) The floor _ adductor longus and oblique muscle medially
E) The floor _ iliopsoas muscle laterally

4560045770 ‫الدكتور يزن ابو غربيه‬


Page 56
Answer : B
discussed before

One is false : (10/2016)


A) femoral artery is lateral to femoral vein in the inguinal area
B) Femoral nerve is present within the femoral sheath
C) Femoral triangle is bounded by sartorius muscle laterally
D) Siatic nerve injury can be avoided by giving im injection on the upper lateral quadrant on
the gluteal area
E) Great saphenous vein drain to femoral vein below the inguinal ligament
Answer : B
Discussed before

A hernia sac containing a Meckels diverticulum is called : (10/2016)


A) Petit hernia
B) Littre hernia
C) Spiglian hernia
D) Richter hernia
E) Grynfeltt hernia

Answer : B
- Spigelian hernia : junction between semilunar line and arcute line
- Pantaloon hernia : direct + indirect hernia
- Richter hernia : knuckle of bowel protrudes into a hernia defect, but only a portion of the
circumference is involved and the bowel lumen remains patent>> lead to gangrenous necrosis f
tissue
- Sliding hernia: any hernia contain abdominal organ
- Internal hernia : bowel trapped after operation with new anatomic relations
- Obturator hernia : hernia in obturator canal >> pressure on obturatr nerve lead pain in medial of
thigh ( howship Romberg sign )

Pediatric GIT surgery


A 3 kg neonate with excessive salivation develops respiratory distress. Attempts to pass an
orogastric tube fail. A chest film shows right upper lobe atelectasis and a gasless abdomen, the
most likely Dx. (7/2018)
a. Proximal esophageal atresia with distal tracheoesophageal fistula
b. H type trachea- esophageal fistula
c. Congenital esophageal stricture
d. Proximal esophageal atresia without a fistula
e. Esophageal atresia with both proximal and distal tracho- esophageal fistula.

Answer: D
The patient has atelectasis with no air enter the stomach that’s mean no fistulae present with
failure to pass the tube that’s mean has esophageal atresia

The treatment of choice for neonates with uncomplicated meconium ileus is; (7/2018)
a. Observation

4560045770 ‫الدكتور يزن ابو غربيه‬


Page 57
b. Emergency laparotomy
c. Intravenous hydration and hypertonic Gastrografin enema
d. Sweat chloride test and pancreatic enzyme therapy.
e. Lower GI endoscopy

Answer: C
We try to dissolve the meconium

Branchial cleft remnants most often present with which of the following clinical problems:
(7/2018)
a. Airway obstruction
b. Hemorrhage
c. Malignancy
d. Pain
e. Infection

Answer: E
Branchial cleft most likely infected in the first year of life after URI

A newborn infant fails to pass meconium in the 1st 48 hours of life and subsequently gets
progressive abdominal distension your order plain films which show distended loops of small
bowel, but no air- fluid levels. The colon is totally decompressed. The transition point is in the
RLQ on exam; he has an anus located in the proper position. The most appropriate next test;
(4/2018)
a. Upper GI endoscopy
b. Barium enema
c. Rectal biopsy
d. Upper GI series
e. Enterolysis

Answer: B
Patient has meconium ileus

The most common type of T.E fistulae: (4/2018)


a. Type A
b. Type B
c. Type C
d. Type D
e. Type E

Answer: C
Type of TE fistulae:
- Type A: Proximal and distal esophageal bud—a normal esophagus with a missing mid-
segment.
- Type B: Proximal esophageal termination on the lower trachea with distal esophageal bud.
- Type C: Proximal esophageal atresia with a distal esophagus arising from the lower trachea
or carina. (Most common )
- Type D : Proximal esophageal termination on the lower trachea or carina with distal esophagus
arising from the carina
Type E /H: A variant of type D: if the two segments of esophagus communicate, this is
sometimes termed an H-type fistula due to its resemblance to the letter H. TEF without EA

4560045770 ‫الدكتور يزن ابو غربيه‬


Page 58
In infantile hypertrophic pyloric stenosis, one of the following is false; (12/2017)
a. The infant usually presents with projectile non bilious vomiting
b. The treatment is surgical by ramstedt’s operation
c. Symptoms commence at age of 3-4 weeks.
d. A palpable olive shaped mass is usually felt in the epigastric region in the classical case.
e. Usually ultrasound is not sufficient for diagnosis.
Answer: E
US is sufficient for diagnosis when shows thickening of pylorus (target like appearance)

All of the following can cause bilious vomiting in an infant or child except; (12/2017)
a. Intestinal malrotation and volvulus
b. Pyloric stenosis
c. Duodenal atresia
d. Small bowel obstruction
e. Ulcerative colitis with colonic obstructive
Answer: B
All other causes is obstruction after ampulla of vater which lead to bilious vomiting except
pyloric stenosis

A 5 year old boy is referred to you as an emergency with right groin and hip. Pain all the
following are important differential diagnosis except: (12/2017)
a. Inguinal hernia
b. Acute appendicitis
c. Testicular torsion
d. Congenital hip abnormality
e. Sigmoid diverticulitis

Answer: E
Diverticulitis comes in old age with history of constipation

All of the following are true regarding Hirschprung disease except; (12/2017)
a. There is no ganglion cells in Anerbach's plexus
b. There is increased incidence of down syn
c. more in boys
d. May be familial
e. On x- ray the affected bowel is narrowed proximally while the normal bowel is dilated distally.

Answer: E
X ray for Hirschprung disease appear as narrow distally with dilated proximally

At what age are intussusceptions most frequent: (12/2017)


a. Newborns
b. Preschool
c. School
d. Adult
e. Infant

Answer: E
80% of intussusception occurs in < 2 years

All of the following are true regarding Hirschsprung disease except ? (7/2017)
A . There is no ganglion cells in Auerbach plexus .

4560045770 ‫الدكتور يزن ابو غربيه‬


Page 59
B . There is an increased incidence of Down syndrome .
C . It is more common in girls .
D. may be familial
E . On X _ ray the affected bowel is narrowed distally , while the normal bowel is dilated
proximall

Answer : C
discussed before

In infantile hypertrophic pyloric stenosis , one of the following statements is false ? (7/2017)
A . The infant usually presents with projectile bilious vomiting .
B . The treatment is surgical by Ramstedt operation .
C . Symptoms commence at the age of 3 _ 4 weeks .
D . A palpable olive shaped mass is usually felt in the classical case

Answer : A
pyloric stenosis present with non bilious vomiting

in the management of pyloric stenosis , which of the following is true ? (4/2018)

A . Acidosis should be corrected first .


B . Chloride level is normal in spite alkalosis .
C . Hypokalemia should be corrected .
D . Usually there is compensatory hypocarbia .
E . Paradoxical aciduria is not a feature

Answer : C
Pyloric Stenosis
Risk factor :
1- whites of Northern European ancestry
2- firstborn
3- males

Clinical pictures :
1- Nonbilious, projectile vomiting
2- Still hungry and desire to feed more
3- Usually age ≥3 weeks (1 week to 5 months)
4- Mild-to-moderate dehydration
5- hypochloremic, hypokalemic metabolic alkalosis
6- Palpation of a firm, movable, 2-cm, olive-shaped, hard mass in midepigastrium
7- left to right peristaltic wave

Diagnosis :
- best test is ultrasound (a target-like appearance in cross-section)

Treatment :
1- Rehydrate, correct electrolytes (NaCl, KCl)
2- Pyloromyotomy

Which of the following is true regarding esophageal atresia and TE fistula ? (7/2016)
A . The most common type is the H type .
B . It is suspected prenatally by Oligohydramnios .
C . In most cases gastrostomy is required before surgical repair .

4560045770 ‫الدكتور يزن ابو غربيه‬


Page 60
D . 20 % of patient have cardiac anomalies .
E . This condition is more in females

Answer : D
TE is part of VACTERL association
V = Vertebral defects
A = Anal atresia (imperforate anus)
C = Cardiac defects (VSD and others)
T = TE fistula
E = Esophageal atresia
R = Renal defects
L = Limb defects (radial)

Thyroid gland
All of the following are true about thyroid gland except; (7/2018)
a. Thyroid gland is one of the largest endocrine glands
b. Thyroid gland consists of 2 lobes right and left isthmus, and pyramidal lobe in 50% of
population.
c. Hormones secreted by the thyroid are not affected by the pituitary Gland
d. Thyroid Gland has 2 arteries and 3 veins
e. Thyroid gland develops at 4 weeks of gestation

Answer: C
Thyroid hormones is controlled by TSH from pituitary

In Graves’s disease all are true except; (7/2018)


a. It is most common cause of thyrotoxicosis
b. More common in women
c. Treatment includes medical , iodine ablation and surgery
d. In Graves disease TSH increased while T4 mark decreased
e. Diarrhea, palpitation, and heat intolerance can be found in Graves’ disease.

Answer: D
In graves there high T4 with low TSH due to negative feedback

All are true matching about thyroid gland cancer except; (7/2018)
a. Papillary carcinoma – best prognosis
b. Follicular carcinoma- blood spread
c. Anaplastic carcinoma – multiple endocrine neoplasia
d. Lymphoma – treatment chemo- radiotherapy
e. Medullary carcinoma – secret calcitonin

Answer: C
Medullary carcinoma is part of MEN IIA, MEN IIB not anaplastic

One is true about hashimoto's thyroiditis; (7/2018)


a. Most common cause of hyperthyroidism worldwide
b. Women to men ratio is 1: 20
c. Autoimmune disease
d. Local enlargement of thyroid gland

4560045770 ‫الدكتور يزن ابو غربيه‬


Page 61
e. Strongly associated with Hla- DR4 and HLA- B4

Answer: C
Hashimoto’s thyroiditis is autoimmune hypothyroidism disease common in female with goiter
Associated HLA-DR5 and HLA-DR3

Treatment of choice of papillary thyroid cancer is; (7/2018)


a. Radiotherapy
b. Chemotherapy
c. Chemotherapy
d. Combined chemo radio therapy
e. Total thyroidectomy and nick dissection

Answer: D
Papillary CA is multifocal cancer spread by lymphatic

Which of the following lab value is most effective at picking up recurrent papillary or follicular
thyroid cancer: (4/2018)
a. TSH
b. Free T4
c. Thyroglobulin
d. Transferrin
e. TRH

Answer: C
After thyroidectomy the patient has to have NO Thyroglobulin unless there is recurrence of the
cancer

All of the following increase suspicion of malignant thyroid nodule except: (12/2017)
a. Age less than 20 years
b. Male gender
c. Age more than 60 years
d. New onset of Hoarseness
e. Pain and tenderness

Answer: E
Subacute thyroiditis is tender and painful

Most common thyroid tumor that spread via lymphatics ? (4/2017)


A . Follicular
B . Anaplastic
C . Medullary
D . Papillary
E . Lymphoma

Answer : D
papillary CA spread by lymphatics
follicular CA spread by blood
anaplastic CA spread by direct extension

All are true about thyroid gland except ? (1/2017)


A . It's composed of two lobes , weighs 25 grams in adult , each lobe 5 cm long , 3 cm wide , 2
cm thick .

4560045770 ‫الدكتور يزن ابو غربيه‬


Page 62
B . Epiglotic cartilage lie just above the gland .
C . It extends from first tracheal ring up to sixth tracheal ring .
D . Infrahyoid muscles lie in front , and sternocleidomastoid muscle to the side of the gland .
E . It has follicles , follicular cells , and parafollicular cells .

Answer : B
epiglotic cartilage away from thyroid

The preferred operation for initial management of a thyroid nodule that is considered
suspicious of malignancy by FNAB is ? (1/2017)
A. Nodule excision
B. Partial lobectomy
C. Total thyroidectomy
D. Near total thyroidectomy
E. Total lobectomy and isthmusectomy

Answer : B
since there is suspicious of malignancy we do partial lobectomy
if it is follicular CA we do hemi thyroidectomy then follow up
if it is papillary CA we do total thyroidectomy

Hyperthyroidism can be caused by all except ? (1/2017)


A. Medullary thyroid cancer
B. Gravis disease
C. Plummer disease
D. Struma ovarii
E. Hashimoto disease

Answer : A
medullary thyroid CA secrete calcitonin

The principal blood supply to the parathyroid glands is by ? (1/2017)


A . Superior thyroid arteries
B . Inferior thyroid arteries
C . Thyroid ima arteries
D . Parathyroid arterial branches from external carotid arteries .
E . Highly variable

Answer : B
thyroid takes its blood supply from inferior and superior thyroid arteries but majorly from
inferior thyroid artery

Most common cause of goitrous hypothyroidism in adults ? (1/2017)


A. Hashimoto disease
B. Gravis disease
C. Riedel thyroiditis
D. De quervain thyroiditis
E. Iodine deficiency

Answer : A
some books said it is hashimoto’s and the other said it is iodine deficiency

4560045770 ‫الدكتور يزن ابو غربيه‬


Page 63
The most precise diagnostic screening procedure for differentiating benign thyroid nodules
from malignant ones is ? (1/2017)
A. Thyroid ultrasonography
B. Thyroid scintision
C. Thyroid hormone suppression
D. FNAB
E. CT scan

Answer : D
as we send the sample for cytology

The preferred operation for initial management of a thyroid nodule that is considered
suspicious of malignancy by FNAB is : (10/2016)
A) Nodule excision
B) Partial lobectomy
C) Total lobectomy and isthmusectomy
D) Total thyroidectomy
E) Near total thyroidectomy

Answer : B
Discussed before

A 36 years old woman , 20 weeks pregnant , was in investigating a 1.5 cm right thyroid nodule
, fine needle aspiration is consistent with Papillary neoplasm , which of the following methods
of treatment is contraindicated : (10/2016)
A) Right thyroid lobectomy
B) Subtotal thyroidectomy
C) Total thyroidectomy
D) Total thyroidectomy with lymph node dissection
E) Radioactive iodine ablation of thyroid gland

Answer : E
as pt is pregnant radioactive iodine ablation effect thyroid of fetus

Hyperthyroidism can be caused by all of the following except : (10/2016)


A) MTC
B) Graves disease
C) Plummer disease
D) Struma ovarii
E) Hashimoto disease

Answer : A
- medullary thyroid CA secrete calcitonin
- hashimoto’s can present initially with hyperthyroidism

Most common cause of Goitrous hypothyroidism in adults is : (10/2016)


A) Hashimoto disease
B) Graves disease

4560045770 ‫الدكتور يزن ابو غربيه‬


Page 64
C) Riedels thyroiditis
D) De Quervain thyroiditis
E) Iodine deficiency

Answer : A
discussed before
The most precise diagnostic screening procedure for differentiating benign thyroid nodules
from malignant ones is : (10/2016)
A) Thyroid ultrasonography
B) Thyroid scintiscan
C) Thyroid hormone suppression
D) FNAB
E) CT scan

Answer : D
as we send the sample for cytology to see type of cells malignant or benign

Factors related to increased risk of thyroid cancer include all of the following except ? (7/2016)
A. Family history
B . Hx of radiation
C . Smoking
D . Female
E . Low iodine diet

Answer : E
low iodine diet lead to goiter but not risk factor for thyroid CA

Parathyroid Gland
A 56 years old man presents with constipation, abdominal pain, kidney stones, depression,
which of the following is the most possible causes (7/2018)
a. Hypercalcemia
b. Hypokalemia
c. Hypothyroidism
d. Dehydration
e. DM

Answer: A
Clinical pictures of hyperCa:
1- CNS: Coma, confusion, convulsions
2- GIT: constipation, anorexia, nausea, vomiting, pancreatitis, ulcer disease
3- Renal: polyuria, polydipsia, nephrogenic diabetes insipidus, kidney stones
4- CVS: HTN, ECG >> SHORT QT.
- first symptoms of hyperCa is thirst

The earliest sign of Hypocalcemia is usually; (12/2017)


a. Tachycardia
b. Positive chovstek's sign

4560045770 ‫الدكتور يزن ابو غربيه‬


Page 65
c. Carpopedal spasm
d. Hyperthermia
e. Tingling of face , nose , hands

Answer: E
This is the earliest of hypocalcemia

A 54 year old man on renal dialysis is found to have raised serum calcium and PTH . This is
called ? (4/2017)
A . Primary hyperparathyroidism .
B . Secondary hyperparathyroidism .
C . Tertiary hyperparathyroidism .
D . Hypercalcemia of malignancy .
E . Hypoparathyroidism .

Answer : C

Primary Secondary Tertiary pseudohypoparathyroidism


hyperparathyroidism hyperparathyroidism hyperparathyroidism

Etiology Parathyroid adenoma Hypocalcemia : vit D Long –term Failure of target cell to
deficiency , chronic secondary response of PTH ( genetic )
kidney disease hyperparathyroidism
Sign : short 4th, 5th
metacarpals
Short stature
Round face
Mental retardation
Ca High Low High low

Po4 Low Low Variable ( according normal


to KFT )

Calcitonin sometimes elaborated by which one of the following tumors ? (4/2017)

A . Medullary carcinoma of the breast .


B . Medullary carcinoma of the thyroid .
C . Giant cell carcinoma of the pancreas .
D . Carcinoid tumor of appendix .
E . Ganglioneuromas .

Answer : B
medullary CA of thyroid :
1- arises from parafollicular cell of thyroid
2- Produces calcitonin
3- component of Men IIA, MEN IIB
4- Treatment: surgery.

4560045770 ‫الدكتور يزن ابو غربيه‬


Page 66
The following causes of Hypercalcemia except ? (4/2017)
A . Sarcoidosis
B . Primary hyperparathyroidism
C . Acute pancreatitis
D . Metastasis bronchial carcinoma
E . Milk Alkali syndrome

Answer : C
in acute pancreatitis there consumption of Ca as it precipitate with fat

The first step in management hypercalcemia is ? (1/2017)


A.IV hydration normal saline 0.9%
B. Hydrocortisone
C. Dialysis
D. Administration of Furosemide
E. Administration of mithramycin

Answer : A
first step IV NS > furosemide > IV bisphosphonates > calcitonin

The principal blood supply to the parathyroid glands is by : (10/2016)


A) Superior thyroid arteries
B) Inferior thyroid arteries
C) Thyroidea ima arteries
D) Parathyroid arterial branches from external carotid arteries
E) Highly variable

Answer : B
All gland supplied by inferior thyroid artery

The first step in management hypercalcemia is : (10/2016)


A) IV hydration normal saline 0.9%
B) Hydrocortisone
C) Dialysis
D) Administration of Furosemide
E) Administration of mithramycin

Answer : A
discussed before

A 54 years old man on renal dialysis is have to raised serum calcium and PTH this is called :
(10/2016)
A) Primary Hyperparathyroidism
B) Secondary Hyperparathyroidism
C) Tertiary Hyperparathyroidism
D) Hypercalcemia on malignancy
E) Hypoparathyroidism

4560045770 ‫الدكتور يزن ابو غربيه‬


Page 67
Answer : C
Discussed before

All of the following are true regarding blood supply to the thyroid , parathyroid glands except
? (7/2016)
A . The parathyroid glands are usually supplied by the superior thyroid arteries .
B . The superior thyroid artery is the first branch of the external carotid artery .
C . The recurrent laryngeal nerves are at risk of injury during ligation of the inferior thyroid
arteries .
D . The external branch of the superior laryngeal nerve is at risk of injury when the superior
laryngeal arteries are ligated .
E . The thyroidea ima artery usually arises from the aorta .

Answer : A
All parathyroid glands supplied by inferior thyroid artery

A patient presents with fatigue and bone pain . Serum calcium level is 11.1 mg/dl , PTH is
elevated . The best next step is ? (7/2016)
A . Operative exploration
B . Neck CT scan
C . Technetium 99m sestamibi imaging
D . MRI
E . Ultrasound scan

Answer : A
- indication of surgery in hyperparathyroidism :
1- symptomatic hyperCa
2- Ca > 11.5 mg /dl
3- renal insufficiency
4- age < 50 years
5- Nephrolithiasis
6- osteoporosis

Which of the following is least likely associated with Hyperparathyroidism ? (7/2016)


A . Cholelithiasis and urolithiasis .
B . Acute pancreatitis .
C . Osteitis fibrosa cystica
D . Diarrhea
E . PUD

Answer : A
gall stones do not occur in hyperparathyroidism

Breast
All of the following are risk factors for breast cancer except; (7/2018)
a. Obesity

4560045770 ‫الدكتور يزن ابو غربيه‬


Page 68
b. Low bone mineral density
c. High breast density
d. Smoking
e. First full term pregnancy at age of 35 years.

Answer: B
Low bone density has no relation like other choices
Other risk factor of breast cancer:
1- Nulliparity
2- early menarche
3- late menopause
4- obesity
5- smoking
- remember all causes of high estrogen is risk factor for breast cancer

Which of the following is true regarding breast cancer; (7/2018?)


a. The majority of cases with breast cancer are familiar in origin
b. The most important risk factor for developing breast cancer is age.
c. Breast density has no correlation with the risk of breast cancer.
d. Up to 10% of breast cancer is hereditary
e. BRCA 1 mutation is associated with significant increase in breast and endometrial cancer risk.

Answer: C
- small number of breast cancer is familiar up to 10 %
- obesity and breast density increase estrogen which related to breast cancer
- BRCA1 has association with ovarian cancer not endometrial

The most important prognostic factor for breast cancer devoid of systemic metastases is:
(4/2018)
a. Node status
b. Size of tumor
c. Tumor grade
d. Gender of patient
e. Comorbidities

Answer: A
As first lymph node effect in breast cancer is axillary lymph node

While performing a modified radical mastectomy in the patient with breast cancer, you cut the
thoracodorsal nerve, post – operative this patient is most likely to have ; (4/2018)
a. Weak adduction
b. Winged scapula
c. Weak abduction
d. Paralyzed diaphragm
e. Facial drop

Answer: A
- winged scapula in long thoracic injury
- weak abduction in rotator cuff tear
- paralyzed diaphragm in phrenic nerve injury

Regarding breast cancer, all of the following are true except: (12/2017)
a. Tumor spread to the skin means T4 on TNM classification

4560045770 ‫الدكتور يزن ابو غربيه‬


Page 69
b. Eczema around the nipple should always be investigated.
c. True cut biopsy is more accurate than fine needle aspiration
d. Bloody nipple discharge is always due to cause
e. Metastases usually involve lung, liver and / or bone.

Answer: D
Intraductal papilloma is the most common cause of bloody nipple discharge

Paget's disease of the breast; (12/2017)


a. Present anywhere on the breast skin
b. Usually bilateral
c. Uncommon breast carcinoma
d. Bad prognosis
e. It's diagnosed by mammogram

Answer: D
Paget’s disease most likely effect nipple on one side diagnosed by skin biopsy
The most malignancy of breast is invasive ductal carcinoma

Regarding breast cancer , all of the following is true except ? (7/2017)


A . Peau d Orange appearance of the breast indicate advanced stage .
B . Eczema around the nipple should always be investigated .
C . Trucut biopsy is more accurate than the needle aspiration ( FNA) .
D . Bloody nipple discharge could be due to a benign cause .
E . Palpable lymph node supraclavicular fossa indicate distant metastasis .

Answer : E
Supraclavicular lymph node metastasis doesn’t indicate distant metastasis

50 years old woman came to your clinic stating that she has a one month history of a palpable
right breast lump . On examination , you can feel a 1 cm mobile , firm nodule at 2 o'clock
position . Your first diagnostic test of choice is ? (7/2017)
A . Excisional biopsy .
B . Incisional biopsy .
C . CT scan .
D . Mammogram .
E . Ultrasound

Answer : D
for female p older than 40 with breast mass do mammogram first but if the female younger
than 40 do U/S

One of the following statement is incorrect ? (4/2017)


A . Elevated CA125 is associated with ovarian carcinoma .
B . Elevated CEA is associated with chondrosarcoma .
C . Elevated CA 15_3 is associated with certain breast cancer .
D . Elevated B_HCG is associated with certain testicular cancer .
E . Elevated CA 19_9 is associated with pancreatic cancer .

Answer : B
CA125 :
1- ovarian CA
2- adenocarcinoma of cervix

4560045770 ‫الدكتور يزن ابو غربيه‬


Page 70
3- endometrium adenocarcinoma
4- GIT CA
5- breast CA
CEA :
1- colorectal CA
2- lung CA
3- breast CA
4- thyroid
5- pancreatic CA
6- liver CA
7- cervix CA
8- bladder CA
CA15-3 :
1- breast CA
2- lung CA
3- ovrian CA

B-HCG :
1- germ cell tumor
2- trophoblastic tumor

CA19-9 :
1- pancreatic CA
2- colorectal CA
3- bile duct CA

All of the following are causes of blood stained nipple discharge except ? (4/2017)

A . Intraductal papilloma .
B . Carcinoma .
C . Ductectasia .
D . Fibrocystic disease .
E . Fat necrosis of breast .

Answer : E
all other causes can do bloody nipple discharge

20 years old woman with a bloody single duct discharge and a palpable mass at the areola no
malignant cells are isolated from discharge the likely diagnosis is ? (1/2017)
A. Ductal carcinoma
B. Duct ectasia
C. Intraductal papilloma
D. Prolactinoma
E. Periductal mastitis

Answer : C
most common cause of bloody nipple discharge is intraductal papilloma

Which of the following is associated with increased risk of development of breast cancer ?
(1/2017)
A. Fibrocystic mastopathy
B. Atypical hyperplasia
C. Severe simple hyperplasia

4560045770 ‫الدكتور يزن ابو غربيه‬


Page 71
D. Papillomatosis
E. Hypertrophy of glandular tissues

Answer : B
atypical hyperplasia is the only malignant the other choices is benign

Post mastectomy and axially clearance for a patient with breast cancer , the patient was found
to have a winged scapula , the most likely cause ? (1/2017)
A . Patient suffered a TIA .
B . Damage to thoracodorsal nerve during surgery .
C . Damage to long thoracic nerve during surgery .
D . Damage to serratus anterior muscle .
E . Dislocated scapula

Answer : C
winged scapula caused by injury to long thoracic nerve

All are risk factors for breast cancer except ? (1/2017)


A. Non smoker female
B. Artificial induced menopause
C. Unbalanced HRT
D. Nulliparity
E. Obesity

Answer : B
most of risk factor of breast cancer related to increase exposure to estrogen but artificial
induced menopause the pt has low exposure to estrogen

A 20 years old with a bloody single duct discharge and a palpable mass at the areola , no
malignant cells and isolated from discharge . The likely diagnosis is : (10/2016)
A) Ductal carcinoma
B) Duct ectasia
C) Intraductal papilloma
D) Prolactinoma
E) Periductal mastitis

Answer : C
most common cause of bloody nipple discharge is intraductal papilloma

Which of the following is associated with increased risk of development of breast cancer :
(10/2016)
A) Fibrocystic mastopathy
B) Atypical hyperplasia
C) Severe simple hyperplasia
D) Papillomatosis
E) Hypertrophy of glandular tissue

Answer : B
since atypical hyperplasia is premalignant

4560045770 ‫الدكتور يزن ابو غربيه‬


Page 72
Post mastectomy and axially clearance for a patient with breast cancer , the patient was found
to have a winging scapula , the most likely cause : (10/2016)
A) Patient suffered a TIA
B) Damage to thoracodorsal nerve during surgery
C) Damage to long thoracic nerve during surgery
D) Damage to serratus anterior muscle
E) Dislocated scapula

Answer : C
Discussed before

Assessment of a breast lump include all , except : (10/2016)


A) Clinical examination
B) Mammography
C) Core biopsy
D) FNA
E) Mastectomy

Answer : E
we need to know the type of lumps before mastectomy

After a modified radical mastectomy , A 45 years old woman reports weakness in her arm . On
exam she has mild weakness . When she internally rotates and adducts her arm . The most
likely injured nerve is : (10/2016)
A) Intercosto_brachial
B) Long thoracic
C) Supraclavicular
D) Medial pectoral
E) Thoracodorsal

Answer : E
thoracodorsal nerve supply the latissimus dorsi which responsible for Depression, adducts,
extends and internally rotates the arm at the shoulder

All of the following are considered risk factors for breast cancer except ? (7/2016)
A . Obesity .
B . Hx of ovarian cancer .
C . Hx of endometrial cancer .
D . Multiparity .
E . Late menopause .

Answer : D
in multiparity there is less estrogen exposure which less risk for breast cancer

4560045770 ‫الدكتور يزن ابو غربيه‬


Page 73
Pituitary, adrenal, MEN
A 20 year old man comes to your office and has a blood pressure 240/120 he states that he gets
headaches sometimes and excessive sweating which of the following tests is best for making
the Dx. (4/2018)
a. Urine cortisol
b. Urine aldosterone
c. Urine VMA and metanephrines
d. Urine epinephrine
e. Urine norepinephrine

Answer: C
The pt has pheochromocytoma the first test to do urine VMA and metanephrines
(catecholamines metabolites)

A 30 years old woman recently diagnosed with MEN 1 syndrome develops confusion
somnolence, any short QT interval on the ECG the next appropriate steps: (4/2018)
a. Alpha – blocker
b. Beta – blocker
c. Calcium channel blocker
d. Intravenous fluid and lasix
e. Dialysis

Answer: D
MEN I: 3P: Parathyroid hyperplasia, pancreatic islet cell tumors, anterior pituitary adenoma
And this pt has symptoms of hyperCa from parathyroid hyperplasia so give IV fluid with lasix

What is the most likely diagnosis in a 45 year old male patient with HTN , development of
facial hair and 7cm suprarenal mass ? (4/2017)
A . Myelolipoma
B . Cushing disease
C . Adrenocortical adenoma
D . Pheochromocytoma
E . Carcinoid tumor

Answer : B
there is symptoms of increase cortisol

The test with highest sensitivity for Pheochromocytoma is : (10/2016)


A) Plasma catecholamine
B) Plasma metanephrine
C) Urinary catecholamine
D) Urinary metanephrine
E) Urinary VMA

Answer : B

Plasma metanephrine testing has the highest sensitivity (96%) for detecting a
pheochromocytoma, but it has a lower specificity (85%). In comparison, a 24-hour urinary
collection for catecholamines and metanephrines has a sensitivity of 87.5%and a specificity of
99.7%

4560045770 ‫الدكتور يزن ابو غربيه‬


Page 74
The initial step in management of SIADH ? (7/2016)
A . Furosemide
B . Hypertonic saline
C . Demeclocycline
D . Isotonic saline
E . Restriction of free water

Answer : E
Treatment of SAIDH :
1- Treat underlying causes
2- Fluid Restriction 800 -1000ml /d
3- Demeclocycline: inhibits ADH
4- V2 receptor blocker: conivapton, tolvaptan
5- Hypertonic saline: Rate of correction 0.5- 1 mmol/L/H

The most common presentation of Pheochromocytoma ? (7/2016)


A . Excess sweating
B . Palpitations
C . Chest pain
D . Headache
E . HTN

Answer : E

The first biochemical abnormality detected in the majority of patient with MEN1 ? (7/2016)
A . Hypoglycemia
B . Hyperchloremia
C . Hypocalcemia
D . Hypercalcemia
E . Polycythemia

Answer : D
MEN I
Parathyroid hyperplasia (most common manifestation)
Pancreatic islet cell tumors (most commonly gastrinoma)
Anterior pituitary adenoma (most common prolactin –secreting tumor)

Note: it is caused by MEN I gen on long arm chromosome 11

MEN IIa
MTC (appear as thyroid nodule)
Pheochromocytoma
Parathyroid hyperplasia

MEN IIb
MTC
Pheochromocytoma
Mucosal neuromas + marfan body habbits

Note: it is caused by RET proto-oncogene on chromosome 10

4560045770 ‫الدكتور يزن ابو غربيه‬


Page 75
Vascular surgery
All of the following are branches of external carotid artery except; (7/2018)
a. Posterior Auricular artery
b. Ascending pharyngeal artery
c. Mandibular artery
d. Maxillary artery
e. Superior thyroid artery

Answer: C
There is no Mandibular artery

A 60 years old man develop TIA's and has a carotid ultra sound that shows an 80% stenosis the
most appropriate next step is: (4/2018)
a. Plavix
b. ASA
c. Warfarin
d. Carotid endarterectomy
e. Observation

Answer: D
Indication of endarterectomy
I- > 75% stenosis
Ii- 70 % stenosis with symptoms
Iii- Bilateral disease and symptoms
IV- > 50 % stenosis and recurring TIA

All of the following are true about TTP except: (4/2018)


a. Enlarged spleen
b. Loss of platelets inhibition
c. Splenectomy is usually indicated
d. The primary therapy is plasmapheresis
e. Thrombosis

Answer: C
TTP is autoantibody-mediated inhibition of the enzyme ADAMTS13 which lead to formation
of small clots in blood vessels

In patient who develop a documented episode of deep vein thrombosis (DVT), the most
significant long term sequel is: (12/2017)
a. Claudication.
b. Recurrent foot infection.
c. Development of stasis ulcer
d. Pulmonary embolization.
e. Diminished arterial perfusion.

Answer: C
The most common complication for DVT is stasis ulcer
But the most significant complication is PE

4560045770 ‫الدكتور يزن ابو غربيه‬


Page 76
Optimal medical therapy in a patient with carotid artery stenosis would include ? (7/2017)
A . Smoking cessation .
B . Antiplatelet agent's .
C . Control HTN .
D . Statins drug's .
E . All are true .

Answer : E
to prevent complication from embolus like stroke ,infarction , retinal vein occlusion

Which of the following is not associated with compartment syndrome ? (4/2017)


A . Disproportionate level of pain .
B . Absent distal pulses .
C . Paresthesia .
D . Heat .
E . Paralysis .

Answer : D
with compartment syndrome there is clod not heat

Which of the following is not an increased risk for venous Thromboembolism ? (4/2017)
A . Active cancer .
B . Age greater than 60 .
C . Thrombophilia .
D . Obesity BMI > 30 .
E . Surgery to excise scalp lipoma .

Answer : E
all other choices are risk factor for DVT

All of the following are branches from the external carotid artery except ? (4/2017)
A . Superior thyroid artery
B . Ascending pharyngeal artery
C . Lingual artery
D . Inferior thyroid artery
E . Facial artery

Answer : D
inferior thyroid artery arise from thyrocervical trunk

Which of the following is true regarding the femoral pulse ? (4/2017)


A . It is usually felt at the midpoint of inguinal ligament .
B . The femoral nerve is medial to the femoral pulse .
C . The femoral pulse is lateral to the deep inguinal ring .
D . The femoral pulse is felt at the mid inguinal point .
E . A Radio_femoral delay is usually felt in aortic stenosis

Answer : D
it lies midway between the anterior superior iliac spine and the symphysis pubis.
The common femoral artery gives off the profunda femoris artery and becomes the superficial
femoral artery to descend along the anteromedial part of the thigh in the femoral triangle. It
enters and passes through the adductor canal, and becomes the popliteal artery as it passes

4560045770 ‫الدكتور يزن ابو غربيه‬


Page 77
through an opening in adductor magnus near the junction of the middle and distal thirds of the
thigh

All are true regarding Heparin except ? (4/2017)


A . It's heterogeneous mixture of sulfated polypeptides .
B . Potentiates the actions of antithrombin 3 .
C . Has a half life of a 90 minutes .
D . Can be reversed by protamine sulphate .
E . Can induce an HIT

Answer : A
is a heterogenous preparation of anionic, sulfated glycosaminoglycan polymers

A palpable radial pulse indicate a systolic blood pressure greater than ? (4/2017)
A . 40 mmhg
B . 60 mmhg
C . 80 mmhg
D . 100 mmhg
E . 120 mmhg

Answer : C
palpable radial puls indicate SBP > 80 mmhg ]
palpable femoral pulse indicate SBP > 70 mmhg
palpable carotid pulse indicate SBP > 60 mmhg

Capillary refill time is elicited by ? (4/2017)


A . Pressing over the skin for 3 seconds , normally return perfusion by 2 seconds .
B . Pressing over the skin for 5 seconds , normally return perfusion by 2 seconds .
C . Pressing over the skin for 5 seconds, normally return perfusion by 5 seconds .
D . Pressing over the skin 10 seconds , normally return perfusion by 5 seconds .
E . Pressing over the skin for 2 seconds , normally return perfusion by 5 seconds .

Answer : B
according to many sources this is the capillary refill time

Allen's test is used to examine ? (1/2017)


A. Suppurative tenosynovitis
B. Thoracic outlet syndrome
C. Dislocated HIP
D. Nerve supply the upper limb
E. Blood supply of the hand

Answer : E
Allen test is performed as follow:
1. The hand is elevated and the patient is asked to clench their fist for about 30 seconds.
2. Pressure is applied over the ulnar and the radial arteries so as to occlude both of them.
3. Still elevated, the hand is then opened. It should appear blanched (pallor may be observed at
the finger nails).
4. Ulnar pressure is released while radial pressure is maintained, and the colour should return
within 5 to 15 seconds.
5. If color returns as described, Allen's test is considered to be normal. If color fails to return, the
test is considered abnormal and it suggests that the ulnar artery supply to the hand is not
sufficient. This indicates that it may not be safe to cannulate or needle the radial artery.

4560045770 ‫الدكتور يزن ابو غربيه‬


Page 78
All are true about caludication pain except ? (4/2017)
A. Increased by night time
B. Not postural
C. ABI < 0.9
D. Increased with exertion

Answer : A
claudication pain is not related to time it is related to exertion ( in early stage )

Allen test is used to examine : (10/2016)


A) Suppurative tenosynovitis
B) Thoracic outlet syndrome
C) Dislocated hip
D) Nerve supply of the upper limb
E) Blood supply of the hand

Answer : E
discussed before

A major bleed in the thigh is best controlled in emergency room by : (10/2016)


A) Tourniquet
B) Direct compression by hand
C) Exploration under local anesthesia and clamp
D) Exploration and ligation under local anesthesia
E) Send to operating room for exploration under General anesthesia

Answer : B

Capillary refill time is elicited by : (10/2016)


A) Pressing over the skin for 3 seconds , normally return of perfusion by 2 seconds
B) Pressing over the skin for 5 seconds , normally return of perfusion by 2 seconds
C) Pressing over the skin for 5 seconds , normally return of perfusion by 5 seconds
D) Pressing over the skin for 10 seconds , normally return of perfusion by 5 seconds

Answer : B
Discussed before

Which of the following is not associated with compartment syndrome : (10/2016)


A) Disproportionate level of pain
B) Absent distal pulse
C) Paresthesia
D) Heat
E) Paralysis

Answer : D
discussed before

4560045770 ‫الدكتور يزن ابو غربيه‬


Page 79
Coarctation of the aorta most commonly occur in ? (7/2016)
A . Ascending thoracic aorta .
B . Transverse thoracic aorta .
C . Thoracic aorta just proximal to left Subclavian artery .
D . Thoracic aorta just distal to left Subclavian artery .
E . Descending thoracic aorta just above diaphragm .

Answer : D
90% of coarctation of aorta just below origin of left subclavian artery at origin of ductus
arteriosus (juxtaductal coarctation)

A 32 year old woman notes that her hands become cold and painful when exposed to cold
temperature . Change in color ( pale _ blue _ red ) , vascular pulse is normal , the best next step
in management ? (7/2016)
A . Upper limbs sympathectomy
B . Prostaglandins
C . Fluoxetine
D . Arteriography
E . Diltiazem

Answer : E
the pt has raynaud syndrome
- Episodes of pallor or cyanosis in response to cold or emotional stimuli
- After rewarming the hands >> the blood flow will rebound and skin will appear reddened.
Etiology: vasoconstriction of blood vessels that result in reduced blood flow, cyanosis is
created by deoxygenation of slow- flowing blood.
Clinical picture:
1. Sudden attack of pallor or cyanosis triggered by rapid changes in ambient temperature.
2. Cold sensitivity of other area ear, nose, lower extremities.
Diagnosis:
1. Nailfold capillarscopy test : to differentiate between 1ry and 2nd Raynaud.
Treatment :
- Penicillamine
- CCB : nifedipine
- ACE inhibitor for HTN.

Abdominal compartment syndrome should be suspected when bladder pressure exceed ?


(7/2016)
A . 10 cm H2o
B . 25 cm H2o
C . 35 cm H2o
D . 40 cm H2o
E . 50 cm H2o

4560045770 ‫الدكتور يزن ابو غربيه‬


Page 80
Answer : B

Which of the following is the most common risk factor for spontaneous venous
thromboembolism ? (7/2016)
A . Antithrombin 3 deficiency
B . Factor 5 Leiden
C . Protein C deficiency
D . Protein S deficiency
E . Antiphospholipid syndrome

Answer : B

Trauma patients are at highest risk of venous thromboembolism is ? (7/2016)


A .Head trauma
B .Femur fracture
C .Pelvic fracture
D .Splenectomy
E .Spinal cord injury

Answer : C
- pelvic fracture lead to DVT in iliac vein which more common
- femur fracture lead to fat embolism
- other causes can do DVT but less common

Lung
All of the following signs suggest a tension pneumothorax except: (7/2018)
a. Hypotension
b. Tachycardia
c. Dyspnea
d. Tracheal deviation
e. Subcutaneous emphysema

Answer: e
If there is subcutaneous emphysema that means air is not trapped in thorax

Most common site of needle thoracostomy in management of tension pneumothorax is:


(7/2018)
a. 1st intercostals space mid clavicular line
b. 2ed intercostals space mid axillary line
c. 2ed intercostals space mid clavicular line
d. 5th intercostals space mid clavicular line
e. 5th intercostals space anterior axillary line

Answer: C
And chest tube in 5th intercostal space mid axillary

Which lung cancer is most likely to be associated with a paraneoplastic syndrome? (4/2018)
a. Small cell lung cancer
b. Adenocarcinoma
c. Squamous cell carcinoma

4560045770 ‫الدكتور يزن ابو غربيه‬


Page 81
d. Large cell cancer
e. mesothelioma

Answer: A
Paraneoplastic syndrome (SIAD, DI, Eton Lambert syn …) common with small cell CA

A 25 years old then and tall man suddenly develops left sided chest pain cough and dyspnea
provisional diagnosis is; (12/2017)
a. Atelectasis
b. Pneumothorax
c. Pulmonary embolism
d. Myocardial infarction
e. Brocho pneumonia

Answer: B
This young thin tall man commonly present with spontaneous pneumothorax due to bleb
rupture

A 41 year old gentleman came to your clinic complaining of respiratory symptoms . Your
differential diagnosis is pleural effusion versus pneumothorax . Which of the following
physical findings will best help you differentiate between these two possibilities ? (7/2017)
A . Respiratory rate 33 breaths / minute .
B . Agitation due to respiration distress .
C . Decreased breath sounds over the affected hemithorax .
D . Dullness to percussion over the affected hemithorax .
E . Cough .

Answer : D
percussion will help to differentiate between pneumothorax and pleural effusion
pneumothorax = resonant on percussion
pleural effusion = dull on percussion

A 39 year old patient presented to you at the emergency room after chest trauma . Which of the
following scenarios is most consistent with left tension pneumothorax ? (7/2017)
A . Hypotension , distended neck veins , midline trachea and muffled heart sounds .
B . Hypotension , open 4 cm 215 , 4 cm lateral chest wound .
C . Hypotension , diminished breath sound on the left tracheal opacification of the left
hemithorax .
D . Diminished left sided breath sounds , tracheal deviation to right .
E . Hypotension , adjacent three fractured in two places .

Answer : D
in tension pneumothorax the air trapped in the chest lead to decrease breath sound in the same
side and tracheal deviation to other side

A 34 years old male patient presented after being in RTA with severe shortness of breath .
Examination revealed a deviated trachea to the left side . Absent breath sounds on the right side
of the chest . The best initial management option for this patient is ? (1/2017)

A . Intubate the patient .


B . Insert a chest tube on the right side .
C . Do a chest X_Ray then manage accordingly .

4560045770 ‫الدكتور يزن ابو غربيه‬


Page 82
D . Insert a needle thoracostomy followed by a chest tube in the right side of chest .
E . Insert 2 large bore cannulas and give 1 liter of ringer lactate .

Answer : D
this pt has pneumothorax needs urgently needle thoracostomy then chest tube

Most common cause of SVC syndrome ? (7/2016)


A . Radiation therapy
B . Malignancy
C . Infections
D . Idiopathic
E . Post central line insertion

Answer : B
Over 80% of cases are caused by malignant tumors compressing the superior vena cava. Lung
cancer, usually small cell carcinoma, comprises 75-80% of these cases and non-Hodgkin
lymphoma, most commonly diffuse large B-cell lymphoma, comprises 10-15%

Small cell lung carcinoma is managed by ? (7/2016)


A . Chemotherapy followed by surgery .
B . Surgical resection .
C . Radiation therapy .
D . Combination chemo_radio therapy .
E . Radiotherapy followed by surgery .

Answer : D
- SCLC : chemotherapy + radiation
Note : chemotherapy for SCLC ( cyclophosphamid , doxorubicin , vincristine cisplatin ,
carboplatin , etoposide )
- NSCLC :
stage 1 +2 >> surgical removal
Stage 3 >>surgical removal + chemotherapy
Stage 4 >> non candidates of surgical resection
Note : chemotherapy for NSCLC 2 type >>
i- Platinum based ( cisplatin or carboplatin )
ii- Nonplatinum based ( etoposide , irinotecan , paclitaxel , gemcitabine )

A 60 year old male smoker presents with radicular right arm pain . A chest radiograph reveals a
bulky mass in the posterior apex of the chest in the superior sulcus . All of the following are
commonly associated with this mass except ? (7/2016)
A . Ptosis
B . Right eye miosis
C . Right facial anhidrosis
D . Right eye exophtalmus
E . Non small cell lung cancer

4560045770 ‫الدكتور يزن ابو غربيه‬


Page 83
Answer : D
Horner syndrome : ( ptosis , myosis , anhydrosis ) >> involvement of sympathetic trunk by
lung cancer (Pancoast tumor)
All are true about Bochdalek hernia except ? (7/2016)
A . It is type of congenital Diaphragmatic hernia .
B . Newborn presents with respiratory distress at birth .
C . It is more common in left side .
D . It is associated with intestinal Malrotaion and obstruction .
E . Can result in pulmonary hypoplasia of both lung .

Answer : E
Bochdalek hernia lead to lung hypoplasia on the affected side only not both lung

A Bochdalek hernia is one of two forms of a congenital diaphragmatic hernia, the other form
being Morgagni hernia. A Bochdalek hernia is a congenital abnormality in which an opening
exists in the infant's diaphragm, allowing normally intra-abdominal organs (particularly the
stomach and intestines) to protrude into the thoracic cavity. In the majority of patients, the
affected lung will be deformed, and the resulting lung compression can be life-threatening.
Bochdalek hernias occur more commonly on the posterior left side

Esophagus
The most common location for a boerhaave's syndrome is: (4/2018)
a. The left posterior lower esophagus
b. The left posterior middle esophagus
c. The left posterior upper esophagus
d. The right posterior middle esophagus
e. The right posterior upper esophagus

Answer: A
Boerhaave’s syndrome:
- Full thickness tears in esophagus secondary to extreme vomiting
- Common in alcoholics
- Most common location : left posterolatral aspect of distal esophagus
- Diagnosis : gastrografin esophagram
- Treatment : surgical exploration with debridement of mediastinum and closure of the wound

The typical management finding in patient with achalasia is; (4/2018)


a. Normal LES pressure and absent peristalsis
b. Failure of LES to relax and absent peristalsis
c. Increased of LES pressure and normal peristalsis
d. Increased of LES pressure and high peristalsis
e. Low of LED pressure and Absent peristalsis

Answer: B
In achalasia there is idiopathic neural loss which leads to absent peristalsis with no relaxation
of LES

4560045770 ‫الدكتور يزن ابو غربيه‬


Page 84
One of the following passes through the aortic hiatus in the diaphragm ? (1/2017)
A . Vagal trunks
B . Right phrenic nerve
C . Thoracic duct
D . Esophagus
E. Branches from left gastric artery

Answer : C
aortic hiatus is located at level T12 and thoracic duct with azygos vein pass through it

Which of the following statement about the anatomy of the esophagus is correct ? (1/2017)
A . The cervical esophagus passes behind and to the right of the trachea .
B . The esophagus deviates anteriorly and to the left as it enters the abdomen .
C. The thoracic esophagus enter the posterior mediastinum anteriorly to the aortic arch .
D . The thoracic esophagus passes behind the right mainstem bronchi and the pericardium .
E . The esophagus enters the Diaphragmatic hiatus at the level of T8 .

Answer : B
The cervical esophagus passes behind and to the LEFT of the trachea .
The thoracic esophagus enter the posterior mediastinum POSTERIORLY to the aortic arch .
The thoracic esophagus passes behind the LEFT mainstem bronchi and the pericardium .
The esophagus enters the Diaphragmatic hiatus at the level of T10 .

Which of the following statement about the anatomy of the esophagus is correct : (10/2016)
A) The cervical esophagus passes behind and to the right of the trachea
B) The esophagus deviates anteriorly and to the left as it enters the abdomen
C) The thoracic esophagus enters the posterior mediastinum anteriorly to the aortic arch
D) The thoracic esophagus passes behind the right mainstem bronchi and the pericardium
E) The esophagus enters the diaphragmatic hiatus at the level of T8

Answer : B
- the cervical esophagus passes behind and to the left of trachea
- the thoracic enters the posterior mediastinum posteriorly to the aortic arch
- the thoracic esophagus passes behind the left mainstem bronchi and the pericardium
- esophagus enters the diaphragmatic hiatus at the level of T10

One of the following is an indication for nasogastric tube insertion : (10/2016)


A) Recent esophageal surgery
B) Suspected esophageal perforation
C) PE
D) Perforated stomach
E) Colovesical fistula

Answer : D
we need NG in perforated stomach for gastric decompression

Which of the following statement about esophageal leiomyoma is false ? (7/2016)


A . It is the most common benign esophageal neoplasm .
B . Most common sites middle and lower third

4560045770 ‫الدكتور يزن ابو غربيه‬


Page 85
C . They have risk of malignant degeneration
D . Preoperative endoscopic biopsy should be done .
E . Treatment of choice is endoscopic enucleation

Answer : D
a biopsy should not be performed if it would cause scarring at the biopsy site, which would
hamper definitive extramucosal resection at surgery.\

All are true about Achalasia except ? (7/2016)


A . It can be congenital or acquired .
B . In young patient , myotomy is the best treatment .
C . CCB exacerbate this condition .
D . Caused by neurogenic degeneration .
E . Dysphagia , regurgitation , weight loss .

Answer : C
medical therapy for achalasia
1- CCB : nifedipine
2- nitrates : isosorbide dinitrate and nitroglycerin
3- Botox injection

Neurosurgery
All of the following are branches of lumbar plexus except; (7/2018)
a. Ilioinguinal nerve
b. Lateral femuro cutaneous nerve
c. Posterior femuro cutaneous nerve
d. Femoral nerve
e. Obturator nerve

Answer: C
Posterior femuro cutaneous nerve is a sensory branch of the sacral plexus. It arises from
anterior and posterior divisions of anterior rami of S1, S2 and S3 nerves. It supplies the skin of
the posterior thigh, buttock and the posterior scrotum/labia.

An alcoholic patient was admitted to hospital 30 min after the fight, he was exited there were
ecchymoses and abrasion on left temporal region, and 6 hours later consciousness was 8 points
by Glasgow coma scale, weakness of the right limbs, left Mydriasis what most likely causes;
(7/2018)
a. Subarachnoid hemorrhage
b. Epidural hematoma
c. Intraventricular hemorrhage
d. Focal edema of the brain
e. Alcoholic coma

Answer: B
This pt had epidural hematoma after trauma to middle meningeal artery which lies in temporal
he had drop of consciousness later as he had lucent period (honey moon period) then increase
of ICP with brain herniation lead to limb weakness and Mydriasis

4560045770 ‫الدكتور يزن ابو غربيه‬


Page 86
Circle of Willis composed of all of the following arteries except; (7/2018)
a. Anterior communicating artery
b. Posterior communicating artery
c. Internal carotid artery
d. Middle cerebral artery
e. Anterior cerebral artery

Answer: D
Arteries of circle of Willis:

1- Anterior cerebral artery


2- Anterior communicating artery
3- Internal carotid artery
4- Posterior cerebral artery
5- Posterior communicating artery

A. 25 years old man involved in a motor vehicle accident will open his eyes only to painful
stimuli, does not from words but mumbles and with draw to pain. The Glasgow coma score;
(4/2018)
a. 10
b. 8
c. 7
d. 6
e. 4

Answer: B
Glasgow coma scale:
Eye Opening (E)
4= spontaneous
3 = to sound
2 = to pressure
1= none
Verbal Response (V)
5= orientated
4= confused
3= words, but not coherent
2= sounds, but no words
1= none
Motor Response (M)
6= obeys command
5= localizing
4= normal flexion
3 = abnormal flexion
2 = extension
1 = none

Which is not contributory to Glasgow coma scale; (12/2017?)


a. Obey commands
b. Localize painful stimuli
c. Open eyes to calling
d. Incomprehensible sounds
e. Pupils size

4560045770 ‫الدكتور يزن ابو غربيه‬


Page 87
Answer: E
Discussed before

Which of the following is the most likely diagnosis in head trauma patient who loses
consciousness recovers to have a lucid internal and again loses consciences; (12/2017)
a. Contusion
b. Subdural hematoma
c. Subarachnoid hemorrhage
d. Epidural hematoma
e. Concussion

Answer: D
Discussed before

To allow kidney donation from a young patient intubated in the ICU , a brain death need to
confirmed first , all of the following examination finding are consistent with brain death except
?
A . Dilated non reactive pupils .
B . Absent oculocephalic reflex .
C . Extensor posturing .
D . Absent gag reflex .
E . Absent corneal reflex .

Answer : C
criteria of brain death
6- no respiratory effort
7- absent brain stem reflexes
 fixed mid dilated pupil
 absent corneal reflex
 absent oculovestibular reflex (cold caloric )
 absent oculocephalic reflex ( doll’s eye )
 absent gag and cough reflex
8- no response to deep central pain

25 years old female patient presented to emergency room after being involved in road traffic
accident, 2.5 hours ago as a driver . She complains of right sided headache and can't remember
the accident . Examination showed a deformed fracture on the right side of her skull . You sent
the patient for CT of her head . After CT the patient suddenly became unresponsive . The CT
will most likely show ? (7/2017)
A . Epidural hematoma .
B . Chronic subdural hematoma .
C . Subarachnoid bleeding .
D . Intraventricular bleeding .
E . Diffuse axonal injury .

Answer : A
Discussed before

All of the following are classic signs of basal skull fracture except ? (7/2017)
A . Raccoon eyes .
B . Battle sign .
C . Hemotympanum .

4560045770 ‫الدكتور يزن ابو غربيه‬


Page 88
D . Dilated non reactive pupils .
E . Cerebrospinal fluid rhinorrhea

Answer : D
this sign in for intracranial mass

The GCS for 25 year man involved in RTA in a head on collision with a lorry . On examination
he is unable to obey commands but localizes to pain and is able to speak , but appear confused .
He only opens his eyes to pain ? (4/2017)
A.4
B.6
C.7
D . 11
E . 13

Answer : D
Discussed before

A 45 years old female found to have wasting of intrinsic muscle , of the hand , weakness and
pain in the wrist , most likely nerve injury ? (1/2017)
A . Ulnar nerve
B . Radial nerve
C . Median nerve
D . Axillary nerve
E . Thenar and hypothenar nerves

Answer : A
ulnar nerve supply the medial part of the hand

At which level , end of the spinal cord ? (1/2017)


A. Forth lumbar vertebrae
B . L3_L4 interspace
C . Second lumbar vertebrae
D. T12_L1 interspace
E. S2_S3 interspace

Answer : C
spinal cord ends at L2 but we do LP at L4-L5

The most common type of intracranial hemorrhage ? (1/2017)


A. Extradural
B. Subdural
C. Subarachnoid
D. Intracerebral
E. Intraventricular

Answer : C

At which level , end of the spinal cord : (10/2016)


A) 4th lumbar vertebrae
B) L3_L4 inter space
C) Second lumbar vertebrae

4560045770 ‫الدكتور يزن ابو غربيه‬


Page 89
D) T12_L1 inter space
E) S2_S3 inter space

Answer : C

The most common type of intracranial hemorrhage : (10/2016)


A) Extradural
B) Subdural
C) Subarachnoid
D) Intracerebral
E) Intraventricular

Answer : C
this is the most common intracranial hemorrhage

Carpal tunnel syndrome is due to compression of : (10/2016)


A) Median nerve
B) Ulnar nerve
C) Radial nerve
D) Radial artery
E) Ulnar artery

Answer : A
Carpal tunnel syndrome :

compression of median nerve >> paresthesia in lateral 3 digits causes :


1- Idiopathic : most common cause
2- RA 3-hypothyroidsim 4-pregnancy 5- amyloidosis 6- DM 7- acromegaly 8-
obesity

Neurogenic thoracic outlet syndrome most common affect which nerve ? (7/2016)
A . Radial
B . Ulnar
C . Median
D . Musclocutaneous
E . Axially

Answer : B
ulnar nerve is the inferior division of brachial plexus so it is the most likely to be compressed
Thoracic outlet syndrome (TOS) is a condition in which there is compression of the nerves,
arteries, or veins in the passageway from the lower neck to the armpit. There are three main
types: neurogenic, venous, and arterial.
1- The neurogenic type is the most common and presents with pain, weakness, and occasionally
loss of muscle at the base of the thumb. produced by compression of components of the
brachial plexus nerves
2- The venous type results in swelling, pain, and possibly a bluish coloration of the arm.
produced by compression of the subclavian vein

4560045770 ‫الدكتور يزن ابو غربيه‬


Page 90
3- The arterial type results in pain, coldness, and paleness of the arm. produced by compression of
the subclavian artery
TOS may result from trauma, repetitive arm movements, tumors, pregnancy, or anatomical
variations such as a cervical rib.
The diagnosis may be supported by nerve conduction studies and medical imaging.
Other conditions that can produce similar symptoms include rotator cuff tear, cervical
disc disorders, fibromyalgia, multiple sclerosis, and complex regional pain syndrome.
Initial treatment for the neurogenic type is with exercises to strengthen the chest muscles and
improve posture. NSAIDs such as naproxen may be used for pain. Surgery is typically done for
the arterial and venous types and for the neurogenic type if it does not improve with other
treatments. Blood thinners may be used to treat or prevent blood clots.
The condition affects about 1% of the population. It is more common in women than men and
it occurs most commonly between 20 and 50 years of age.

All of the following are true regarding head injury except ? (7/2016)
A . Epidural hematoma are associated with lucid interval .
B . Subdural hematoma are due to laceration of bridging veins .
C . Epidural hematoma are usually associated with a skull fracture .
D . CT scan shows convex lens hematoma in epidural hematoma .
E . Epidural hematoma generally have a worse prognosis than Subdural hematoma .

Answer : E
subdural hematoma usually associated with brain injury like contusion which may lead to
seizure in future

A 30 year old man is brought into the emergency room after 20 foot fall . He is noted to speak
using inappropriate words . He withdraws and open his eyes to pain only , Glasgow coma scale
score is ? (7/2016)
A .10
B .9
C .8
D .7
E .6
Answer : B
- speak using inappropriate words = 3
- withdraws to pain = 4
- open eye to pain = 2

Kidneys and Bladder


The most sensitive image for urinary tract stone: (7/2018)
a. MRI
b. Ultrasound
c. Intravenous pyelogram
d. Enhanced urinary tract CT
e. Unenhanced urinary tract CT

4560045770 ‫الدكتور يزن ابو غربيه‬


Page 91
Answer: E
Most of kidney stone is calcium content so it is radiopaque appear in X ray or CT scan not with
US but we don’t use contrast in CT scan as the dye will mask the stone

A patient stops making urine after surgery all of the following values are consistent with
Prerenal renal failure except: (4/2018)
a. Urine Na 5 NE g/l
b. Bun/cr ration 35
c. FeNa = 0.3 %
d. Urine osmolality = 200 mos m/l
e. Urine specific gravity = 1.050

Answer: D
Prerenal renal failure:
1. BUN: Cr = 20 : 1
2. Low Una
3. Low fractional excretion of Na.
4. High urine osmolality (> 500).
5. High urine specific gravity ( >1.010)

In a 3 years old boy, painless ball table mass in the right flank, can be all of the following
except ; (12/2017)
a. Wilm's tumor
b. Neuroblastoma
c. Urinary bladder tumor
d. Hydronephrotic kidney
e. Mesoblastic nephroma

Answer: C
Bladder cancer appear in the Suprapubic

Which of the following diagnosis is most likely in a child with HTN and past history of UTI ?
(7/2017)
A . Ureteropelvic junction obstruction .
B . Reflux nephropathy .
C . Wilms tumor .
D . Renal artery stenosis .
E . Posterior urethral valve

Answer : B
reflux nephropathy ( vesicoureteral reflux (VUR)
- Abnormal backflow of urine from bladder to kidney
- Occurs when the submucosal tunnel between the mucosa and detrusor muscle is Short or
absent.
- Predisposition to pyelonephritis >> scarring >> reflux nephropathy (hypertension,Proteinuria,
renal insufficiency to end-stage renal disease [ESRD], impaired kidney growth)

All of the following are true regarding computerized tomography used in urology except ?
(7/2017)
A . CT without contrast is the imaging of choice for acute flank pain .
B . CT with contrast is preferred in renal trauma .
C . MRI is more sensitive in evaluating adrenal masses .

4560045770 ‫الدكتور يزن ابو غربيه‬


Page 92
D . CT is not operator dependent .
E . Uric acid stones are not visualized by CT scan .

Answer : E
uric acid stone doesn’t appear at X ray but appear at CT scan

Acute pyelonephritis is best diagnosed by ? (7/2017)


A . Vesicoureteral reflux .
B . Bacteriuria and pyuria .
C . Delayed renal function .
D . Focal scar in renal cortex .
E . Chills , fever and flank pain

Answer : B
beat diagnosis for UTI is urine analysis (>10 WBCs/HPF, >105 CFU)

A 32 year old man presents with Hematuria . A KUB X_Ray reveals a staghorn calculus in the
right kidney and midstream urine , sample grows proteus . The most likely stone type is ?
(4/2017)
A . Bile pigment stone .
B . Calcium oxalate .
C . Calcium phosphate .
D . Cholesterol stone .
E . Cystine stone .

Answer : C
staghorn stone is formed by magnesium ammonium phosphate and carbonate apatite
crystallization
proteus is urease enzyme producing bacteria change urea to ammonia

A 32 years old man present with Hematuria. A KUB X_Ray reveals a staghorn calculus in the
right kidney and mid stream urine sample grows proteus . The most likely stone type is :
(10/2016)
A) Bile pigment stone
B) Calcium oxalate
C) Calcium phosphate
D) Cholesterol stone
E) Cysteine stone

Answer : C
Discussed before

Prostate and Male Reproductive Organs


All are indications for surgery in being prostatic hyperplasia except; (7/2018)
a. Large prostate 100 grams
b. Refractory urine retention
c. Renal impairment
d. Recurrent UTI
e. Recurrent gross Haematuria

4560045770 ‫الدكتور يزن ابو غربيه‬


Page 93
Answer: A
Indication for TURP:
1. 100 residual volume
2. Acute urinary retention
3. Chronic urinary retention with overflow dribbing
4. Recurrent Hematuria or UTI

At what age is surgical orchiopexy recommended for a child with unilateral undescended tests;
(7/2018)
a. 4-5 years
b. 5-6 years
c. Any time prior puberty
d. 1 year
e. Promptly upon discovery regardless of age

Answer: D
Usually before 2 years as there is high incidence of testicular cancer to develop

A 62 y/o male patient presented with Nocturia (4 time/night) , weak stream, intermittency and
frequency, his direct rectal exam revealed induration of the right lobe of prostate gland , all of
the following statement are true Except : (12/2017)
a. Treat the obstructive symptoms
b. Perform ultrasound for residual urine assessment
c. Request blood sample for prostate specific antigen.
d. Unlikely this patient will need biopsy from prostate.
e. His past history and medication have to be checked.
Answer: D
This patient has urinary obstruction symptoms with lobe induration most likely has prostate
cancer and will need biopsy

57 years old patient presented with nocturia ( 3 time / night ) , weak stream intermittency and
frequency , his digital rectal examination revealed induration of the right lobe of prostate . All
of the following statements are appropriate except ? (7/2017)
A . Treat the obstructive symptoms .
B . Perform ultrasound for residual urine assessment .
C . Blood sample for prostate specific antigen is not usually required .
D . This patient most likely needs prostate biopsy .
E . His past history and medication have to be checked .

Answer : C
this pt has prostate cancer so we need to do PSA

Which of the following statements is true regarding varicocele ? (7/2017)


A . They should be looked for while the patient is supine .
B . They usually result infertility .
C . They may affect sperm motility and count .
D . They result in lower testicular temperature .
E . They should always be required when identified .

Answer : B
Varicocele is Abnormal dilatation of pampiniform plexus (valvular incompetence of spermatic
vein); most on left, rare age <10 years

4560045770 ‫الدكتور يزن ابو غربيه‬


Page 94
- Most common surgically treatable cause of subfertility in men as keep the local temperature
in or around the testicle too high, affecting sperm formation, movement (motility) and function
- present as painless, paratesticular mass “bag of worms”
- treated Surgically if significant difference in size of testes, pain, or if contralateral testis is
diseased or absent

Skin Cancer
All sentences about skin cancer are true except: (12/2017)
a. Basal cell carcinoma are faster growing than squamous carcinoma
b. Malignant melanoma are the most aggressive skin tumors
c. Squamous cell carcinoma is often seen in area exposed to sunlight such as a scalp and face.
d. Rodent ulcer is a basal cell carcinoma
e. Marjolins ulcer is a squamous cell carcinoma

Answer: A
BCC grows in years
SCC grows in month

Which of the following is true presentation for BCC : (12/2017)


a. Painful ulcer is the usual presentation
b. Metastases to lymph node is very rare
c. UV light is not a cause
d. The growth is rapid
e. Treatment of choice is radiotherapy

Answer: B
Basal cell carcinoma don’t reach blood vessels and lymph until late stage

Skin cancer can result from exposure to one of the following ? (7/2017)
A . Microwave .
B . Ionizing radiation .
C . Solar ultraviolet A .
D . X ray .
E . Infrared rays .

Answer : C
it is the most cancerous risk factor for skin cancer

A 84 year old man presents with a small ulcer on the bridge of his nose . It has been present for
a number of months . On examination it has pearly white rolled edges . The type of ulcer is ?
(4/2017)
A . Curling ulcer
B . Cushing ulcer
C . Marjolin ulcer
D . Rodent ulcer
E . Venous ulcer

Answer : D
Rodent ulcer ( basal cell CA ) is the most common type of skin cancer. It often appears as a
painless raised area of skin, which may be shiny with small blood vessels running over it; or it
may present as a raised area with ulceration.

4560045770 ‫الدكتور يزن ابو غربيه‬


Page 95
Basal-cell cancer grows slowly and can damage the tissue around it but is unlikely to spread to
distant areas or to result in death
Risk factors
9- exposure to ultraviolet light ( tanning bed )
10- having lighter skin
11- radiation therapy
12- long-term exposure to arsenic
13- poor immune-system function.
Diagnosis : depends on skin examination, confirmed by tissue biopsy
Treatment :
1- surgical removal : by simple excision or Mohs surgery
2- application of cold
3- topical chemotherapy
4- laser surgery
5- the use of imiquimod

Which of the following skin tumors is the most common in whites ? (4/2017)
A . SCC
B . BCC
C . Malignant melanoma
D . Merkel cell tumor
E . Dermatofibrosarcoma

Answer : B
Discussed before

The most common site for SCC of the lips is ? (4/2017)


A . Upper lip midline
B . Upper lip laterally
C . Lower lip midline
D . Lower lip laterally
E . Oral commissure

Answer : D
this is most common site of SCC in lips

Skin infection and Hypersensitivity


All predispose to necrotizing fasciitis except ? (1/2017)
A.DM
B. Severe head injuries
C. Cortisone therapy
D. Advance malignancy
E. Renal failure

Answer : B
necrotizing fasciitis is common in immunosuppressed patient like : AIDS , steroid dependent pt
.DM

An 18 year old female patient develops urticaria and wheezing after injection of intravenous
contrast for an abdominal CT scan . Her BP 120/60 ,HR 155 / min , RR 30 / min , which of the

4560045770 ‫الدكتور يزن ابو غربيه‬


Page 96
following is the most appropriate immediate therapy ? (1/2017)
A . Intubation
B . Epinephrine
C . B _ blockers
D . Iodine
E . Fluid challenge

Answer : B
treatment of urticaria :
1- avoidance of trigger
2- oral antihistamine
3- Epinephrine + short-burst corticosteroids if sever
4- If H1 antagonist alone does not work >>H1 plus H2 antagonists are effective >> consider
steroids
5- For chronic refractory angioedema/urticaria >> IVIg or plasmapheresis

All predispose to necrotizing fascitis except : (10/2016)


A) Diabetes
B) Severe head injuries
C) Cortisone therapy
D) Advance malignancy
E) Renal failure

Answer : B
Discussed before

Orthopedic surgery
Anterior dislocation of the shoulder is most likely to injury the; (4/2018)
a. Subclavian vein
b. Subclarian artery
c. Axillary artery
d. Axillary nerve
e. Thoracic duct

Answer: D
Axillary nerve damage results in a weakened or paralyzed deltoid muscle the patient will have
difficulty in abducting the arm from approximately 15° away from the body

all sentences About a colles fracture are true except ? (7/2017)


A. A fracture with mild angulation may require closed reduction .
B. Usually produced by fall on out stretched hand .
C. Defined as a fracture of the distal radius with ventral displacement .
D. Occur more frequently in elderly persons as a result of osteoporosis .
E. Could be complicated by neuropathy and volkman ischemic contracture .

Answer : C
- Colles’ fracture : Distal radius fracture with dorsal displacement and angulation, usually from
falling on an outstretched hand (a common fracture!)

4560045770 ‫الدكتور يزن ابو غربيه‬


Page 97
- Smith’s fracture : Reverse Colles’ fracture”—distal radial fracture with volar displacement and
angulation, usually from falling on the dorsum of the hand (uncommon

All sentences about neck of femur fracture are true except ? (7/2017)
A . Typically occur in elderly female patient .
B. Leg may be shortened and internally rotated in displaced fracture .
C . AVN is more common in this place than at other site's in the body .
D . Almost always will require corrective surgery .
E . Prognosis is poor in general

Answer : B
in femur fracture there is externally rotated and shortened limb

A 19 year old man involved in RTA , found to have a femur fracture and crush injury to his
foot . After fixation he becomes dyspneic and hypoxemic . The most likely etiology ? (4/2017)
A . Aspiration
B . Atelectasis
C . Fat embolism
D . Fluid overload
E . Pneumonia

Answer : C
this symptoms typically for PE

A 66 years old woman falls fracturing the surgical neck of humerus . Sex months later she is
unable to abduct her arm's above her hand . Most likely nerve injury ? (4/2017)
A . Median nerve
B . Radial nerve .
C . Ulnar nerve
D . Axillary nerve
E . Musclo_cutaneous nerve

Answer : D
Axillary nerve is responsible for abduction from 18 – 90 degree by deltoid muscle
muscle-cutaneous nerve injury lead to pain ,tingling and decrease sensation over the lateral side
of the forearm
Trauma
The first systemic complication after a major burn injury; (7/2018)
b) Hemodynamic instability
c) Metabolic acidosis
d) Renal failure
e) Hyperkalemia
f) sepsis

Answer: A
All of the other choices happened lately but the first thing happened is loss skin integrity which
leads to dehydration and hemodynamic instability

4560045770 ‫الدكتور يزن ابو غربيه‬


Page 98
A22 year old man suffer a sever pelvic fracture and has hematuria you get a retrograde
cystomethrogram and see an extra peritoneal bladder. Rupture. The most appropriate therapy.
(4/2018)
a. Foley drainage for 7 days
b. Exploratory laparotomy
c. Cystectomy
d. Observation
e. Cystoscopy and repair.

Answer: A
In patient with hematuria after pelvic trauma don’t use Foley catheter. First do retrograde
cystourethrogram if the bladder rupture extra peritoneal insert Foley for 1 month and it will
heal
But the bladder ruptures intra peritoneal do laparotomy

A 26 year old woman is involved in a sever MVA she is Bp80/40 HR 120 and has distended
abdomen. You give her 2 liter ringer lactate and then start blood transfusion, however she
remain hypotensive, the most appropriate next step is: (4/2018)
a. Abdomen CT scan
b. Abdomen angiography
c. Diagnostic peritoneal lavage or FAS
d. Exploratory laparotomy
e. Abdomen x- ray

Answer: C
This pt is suspect with internal hemorrhage so we have to do FAST test to make about the
hemorrhage then laparotomy
CT scan done for stable pt

65 y/o gentleman presented to you at the emergency room after road traffic accident
complaining of left chest pain, his vital signs where Bp110/50, Hr 100 / RR20 Glasgow coma
scale 14/15 on examination, he was found to have diminished breath sounds on the left side
chest x- ray demonstrated pacification of the left hemi- thorax what is the most likely Dx.
(12/2017)
a. Left hemothorax
b. Left pneumothorax
c. Cardiac tamponade
d. Splenic injury left tension pneumothorax.
Answer: A
this pt has hypovolemic condition which most likely bleeding in the chest as appear whitish on
X ray ( not like pneumothorax )

You were the night shift doctor at the Emergency room when a 50 year old gentleman
presented to you after a trauma to his abdomen. His vital signs were Bp110/60 HR 90 RR17,
Glasgow com scale 15/15 Physical exam his abdomen is soft but tender on the right upper
quadrant. What will be your best next step: (12/2017?)
a. Diagnostic Assessment lavage
b. Focused Assessment by sonography test
c. ICU admission
d. Call the surgeon for immediate laparotomy
e. CT scan of the abdomen

4560045770 ‫الدكتور يزن ابو غربيه‬


Page 99
Answer: E
As pt is stable with suspicion for internal bleeding do CT scan if unstable do FAST

All sentence about trauma are true except; (12/2017)


a. The term triage is generally used in acute trauma life support (ATLS )
b. The rationale behind triage is for the benefit of the majority
c. The role of the designated triage person is only to triage and not to treat
d. The triage process must be constantly repeatedly
e. Coma correspond to a Glasgow coma scale of more than 8

Answer: E
Coma patient has less than 8 in Glasgow coma scale and consider intubation for him

All sentences about burn are correct except: (12/2017)


a. Burn area can be calculated by the rule of nines.
b. Fluid resuscitation is calculated based on burn area.
c. I.V fluid resuscitation in necessary in child with 10% partial thickness burn.
d. Prophylactic intubation is rarely indication in inhalation burn.
e. Full thickness burns extends into deep reticular dermis, yellow white appearance, fairly dry
texture, painful, needs 3-8 weeks, for healing.

Answer: E
Full thickness it’s the 3rd degree burn not painful as nerve ends is destroyed and need graft for
healing
1st degree: limited to dermis with no blister, red and painful, heal without scar
2nd degree: extend to dermis with blister, red and painful, may heal with scar
3rd degree: entire dermis destroyed, no pain a nerve ending is damaged, Need grafting

At 2 day's after a motor vehicle crash , an otherwise healthy 30 years old man is recovering in
the ICU from rib fractures , pulmonary contusion , and a liver laceration . He complains of
increasing right upper quadrant pain and vomits bright red blood . Which of the following
approaches is best ? (7/2017)
A . Operative exploration and liver resection .
B . Hepatic artery ligation .
C . Angiographic arterial embolization .
D . ERCP with sphincterotomy .
E . Coagulopathy workup .

Answer : A
rib fracture lead to liver laceration

The most reliable indicator of the adequacy of burn resuscitation is ? (4/2017)


A . CVP .
B . PCWP .
C . Urine output .
D . Blood pressure .
E . Heart rate per minute .

Answer : C
this indicator of good resuscitation

4560045770 ‫الدكتور يزن ابو غربيه‬


Page 100
Regarding abdominal trauma in pediatric age group , which of the following is true ? (4/2017)
A . Clinical examination , stabilization and resuscitation are not of priority .
B . Ultrasound is the diagnostic tool of choice .
C . Blood transfusion should be started immediately .
D . Peritoneal lavage is contraindicated .
E . Splenectomy is not a routine management for all types of Splenic injuries .

Answer : E
we can do splenoraphy in some cases
ct scan and peritoneal lavage is diagnostic tools for spleen hemorrhage

One is true regarding electric burns ? (4/2017)


A . Injuries are generally more superficial than those with thermal burns .
B . Intravenous fluid replacement is based on the percentage of body surface area burnt .
C . Electric burns often result in transient traumatic optic neuropathy .
D . Evaluation for fracture of the other extremities and visceral injury is indicated .
E . Cardiac conduction abnormalities are unlikely .

Answer : D
- High-voltage electrical burns are always deeper and worse than they appear to be.
- Massive debridements or amputations may be required.
- Additional concerns include myonecrosis induced acute kidney injury, orthopedic injuries
secondary to massive muscle contractions (e.g., posterior dislocation of the shoulder,
compression fractures of vertebral bodies)
- late development of cataracts and demyelinization syndromes
- Of course cardiac electrical integrity and function must be evaluated
.

60 kg woman involved with extensive burns . On physical exam found to have pink to red skin
colour , slightly swelling and pain , dry skin on the head . Her left arm and leg have dry white
painless burns and her anterior torso has Erythematous painful burns with blistering. How
much fluid she require ? (4/2017)
A . 2500 ml
B . 10800 ml
C . 8640 ml
D . 12960 ml
E . 17280 ml

Answer : B
left leg =18%
left arm = 9%
anterior torso = 18%
first degree isn’t counted in parkland formula
4*60 kg *45 = 10800

4560045770 ‫الدكتور يزن ابو غربيه‬


Page 101
All are true about burns except ? (1/2017)
A . Blisters , hair follicles presents _ super facial partial thickness .
B . No pain , hard leathery eschar , black grey_ white color _ full thickness .
C . Best IV solution is normal saline .
D . Circumferential burns require emergency Escharotomy .
E . Prognosis best determined by burn size ( TBSA )

Answer : C
best IV solution is RL

First degree burns involving the both hands , second degree burns involving the entire head
and neck , third degree involving the entire back and half of the chest , would be considered :
(10/2016)
A) 12% TBSA
B) 27% TBSA
C) 36% TBSA
D) 40% TBSA
E) 45% TBSA

Answer : C
we don’t count 1st degree
Head =9%
back = 18 %
half of chest = 9 %
totally = 36%

According to Parkland formula , the amount of crystalloid fluids , that should be administered
to the first 8 hours to a 70 kg man with a 15% TBSA first degree burns , 50 % TBSA second
degree burn , 20% TBSA third degree burn : (10/2016)
A) 5000 ml
B) 6500 ml
C) 7350 ml
D) 9800 ml
E) 11900 ml

Answer : D
parkland = 70 kg * 70% ( don’t count 1st degree ) * 4 = 19600
half of it in the first 8 hour = 9800

Shock
A 60 years old woman with no past medical problems presents to the E.R with jaundice RUQ
rain, and fever her Bp 80/40 HR120 the most appropriate next step: (4/2018)
a. Emergent cholecystectomy
b. Emergent ERCP
c. Emergent PTC placement

4560045770 ‫الدكتور يزن ابو غربيه‬


Page 102
d. Volume resuscitation and IV Antibiotic
e. Emergency laparotomy

Answer: D
The patient has septic shock treated first

In a patient with hypovolemic shock, which cannula size would you ideally choose for
infusion: (12/2017?)
a. 19 G
b. 21 G
c. 14 G
d. 26 G
e. 30 G

Answer: C
In shock pt we need large bore cannula and the largest one for 14 G

Routine tracheostomy , with few exceptions is typically performed at the level of ? (7/2017)
A . Cricoid cartilage .
B . Cricothyroid membrane .
C . Cricoid and first ring .
D . Second ring and third ring .
E . Fifth and sixth tracheal ring

Answer : D

All of the following are true about the assessment of bleeding except ? (7/2017)
A . Taking blood sample at the time of bleeding gives an accurate level of haemoglobin .
B . Hypovolemia increase the risk of tissue hypoxia and multi organ failure .
C . Blood loss could be estimated from the size of blood clot and the weight of swabs used .
D . Any thirsty patient should be considered Hypovolemic .
E . Arterial blood gas is a quick method to measure hemoglobin

Answer : A

All of the following sentences are true about pathophysiology of shock except ? (7/2017)
A . Shock means inadequate oxygen delivery to meet metabolic demands .
B . Shock may occur without loss in the circulating volume .
C . Severe infection may lead to decreased cardiac preload .

4560045770 ‫الدكتور يزن ابو غربيه‬


Page 103
D . Serum lactate decreases in shock .
E . The function of phagocytes is decreased in shock

Answer : D
LDH increase not decrease with shock

Which of the following supports the diagnosis of Hypovolemia ? (4/2017)


A . Urine sodium 28 meq/L .
B . Urine chloride 15 meq/L .
C . Fe Na < 1 .
D . Urine / serum creatinine ratio of 20 .
E . Urine Osmolarity of 350 mosm/ kg

Answer : C
Prerenal Azotemia criteria :
1) BUN: Cr = 20 : 1
2) Low Una
3) Low fractional excretion of Na.
4) High urine osmolality (> 500).
5) High urine specific gravity ( >1.010)

What is the first parameter to change in early hypovolemic shock ? (4/2017)


A . Systolic blood pressure .
B . Pulse rate .
C . Diastolic blood pressure .
D . Respiratory rate
E . Level of consciousness

Answer : B
this is the early sing of hypovolemic shock

Which of the following is not true regarding sepsis ? (4/2017)


A . Tachycardia and / or tachypnea may be a sign .
B . Sepsis is defined as systemic inflammatory response syndrome in the presence of infection
C . Leukocytosis or leukopenia may be a sign .
D . Sepsis and hypotension is defined as septic shock .
E . Temperature greater than 38 or less than 36 may be a sign

Answer : D
septic shock = sepsis + hypotension + hypoperfusion

A 20 year old man involved in RTA sustains bilateral fractured femurs . On assessment , he is
anxious and confused , with a pulse of 130 / min and BP of 70/50 mmhg . Urine output over 1
hour is 10 ml . This patient has hemorrhagic shock type ? (4/2017)
A. Class 1
B. Class 2
C. Class 3
D. class 4
E. class 5

Answer : C

4560045770 ‫الدكتور يزن ابو غربيه‬


Page 104
Stage of hypovolemic shock

Stage 1 : Mild anxiety, normal vital signs


(<15% or 750 cc
blood loss)
Stage 2 : Normal systolic BP with decreased pulse
(15%–30% or pressure, tachycardia, tachypnea, anxiety
750– 1500 cc
blood loss
Stage 3 : Tachycardia (heart rate>120), tachypnea (respiratory
(30%–40% or rate>30), decreased systolic BP, decreased pulse
1500– 2000 cc
blood loss) pressure, confusion
Stage 4 : Decreased systolic BP, tachycardia (heart rate >140),
(>40% or >2000 tachypnea (respiratory rate >35), decreased pulse
cc blood loss
pressure, confused and lethargic, no urine output

Which of the following is not true regarding sepsis : (10/2016)


A) Tachycardia and / or tachypnea may be a sign
B) Sepsis is defined as systematic inflammatory response in the presence of infection
C) Leucocytosis or Leukopenia may be a sign
D) Sepsis and hypotension despite IV fluid is defined as septic shock
E) Temperature greater than 38 or less than 36 may be a sign

Answer : D
discussed before

In the management of hemorrhagic shock , the best clinical sign of successful fluid
resuscitation is : (10/2016)
A) Increase in blood pressure
B) Increase in urine output
C) Increase in arterial oxygenation
D) Decrease in thirst
E) Decrease in tachycardia

Answer : B

when urine increase that’s mean adequate renal perfusion with ends of sympathetic
stimulation and the end of hypovolemic shock

Which of the following is an appropriate definition of shock ? (7/2016)


A . Low blood pressure
B . Low cardiac output
C . Low circulating volumes
D . Inadequate tissue perfusion .
E . Abnormal vascular resistance .

Answer : D
this is the definition of shock

4560045770 ‫الدكتور يزن ابو غربيه‬


Page 105
Surgical nutrition

Wound healing
All of the following are clean or clean contaminating surgery except; (7/2018)
a. Hernia repair
b. Thyroidectomy
c. Laparoscopic cholecystectomy
d. Bowel perforation
e. Radical mastectomy

Answer: D
Bowel perforation is dirty wound

The most predominant cell type in a 7 day old wound is: (4/2018)
a. PMNs
b. Macrophages
c. Lymphocytes
d. Fibroblasts
e. Platelets

Answer: D
PMN: hours – 2 days
Macrophages: 2-4 days
Lymphocytes: has no Role
Platelets: 1st hour

All are true about Keloid scar except: (4/2018)


a. They are confined to the original scar area.
b. Can be treated with silicon, steroid, or radiotherapy
c. Occur mainly on the earlobe, shoulder sterna notch, rare across joints.
d. More common in dark skin
e. More common in females than males.

Answer: A
It is the hypertrophic scar which confined to original scar while Keloid extend beyond the
original scar

All sentences about wound healing are correct except: (12/2017)


a. Presence of a foreign body delays the wound healing
b. The final strength of a healed wound is greater than that of the surrounding skin.
c. the surgical wound of thyroid operation is considered as clean wound
d. Staphylococcus aureus is the most common cause of would infection following surgery.
e. Hand wash and hygiene decrease community and hospital acquired wounds infections

Answer: B
The maxim tensile strength of the wound is 80% of the original skin

4560045770 ‫الدكتور يزن ابو غربيه‬


Page 106
Which of the following statement regarding the differentiation of Keloid scars from
Hypertrophic is false : (12/2017)
a. Keloid scars are most often seen in dark skin people
b. Hypertrophic scars respond better to treatment there Keloid scars.
c. Keloid scars occur when scar cross joints at right angle.
d. Keloid scars outgrow the wound area.
e. Keloid scars have a familial predisposition

Answer: C
Most common site of Keloid is sternum, ear, and deltoid

All are true statement except ? (7/2016)


A . The most predominant cell at day 3_4 after a wound is macrophages .
B . The most predominant cell in the first 24 hours of the wound is PMN .
C . The order of cell arrival in wound is macrophages , platelets , PMNs , fibroblasts .
D . The most predominant cell type in a 7 day old wound is fibroblasts .

Answer : C
the order of cells in wound healing
1- platelet arrive the wound in the first hours
2- PMN is the predominant cell in 0-2 days
3- macrophages is the predominant cell in 3-5 days
4- fibroblasts is the predominant cell > 5 days

The first step in process of hemostasis after vessel injury is ? (7/2016)


A . Platelet adherence
B . Platelet aggregation
C . Vasoconstriction
D . Release of thromboxane
E . Thrombus activation

Answer : C
step of hemostasis :
1- vasoconstriction
2- platelet adherence
3- platelet aggregation
4- coagulation

The most useful laboratory test to assess risk of bleeding and response to therapy in patient
with Uremia is ? (7/2016)
A . Bleeding time
B . Platelet count
C . INR
D . PTT
E . Thrombin time

Answer : A
- as uremia lead to inhibition of platelet function and impaired platelet-vessel wall interaction

4560045770 ‫الدكتور يزن ابو غربيه‬


Page 107
- PTT , PT , INR , thrombin time reflect the clotting factors not platelet

Head and Neck


A 50 years old man has a mass 1 cm anterior to the ear the mass causes him pain and facial
drop ct of the head shows mass in both the deep and superficial part of the parotid gland, the
most likely Dx : (4/2018)
a. Mucoepidermoid carcinoma
b. Adenoid cystic carcinoma
c. Pleomorphic adenoma
d. Warthin tumor

Answer: B
Mucoepidermoid carcinoma is the most common malignant in parotid but the adenoid cystic
carcinoma present with early neural invasion

All sentences about sialolithiasis are true except ? (7/2017)


A . Parotid gland is the most common site of salivary calculi .
B . Salivary calculus contains debris bacteria , mucous , magnesium and phosphates .
C . Easily demonstrated by x Ray .
D . Most occur in middle aged adults .
E . The main symptoms are pain and swelling before and during eating .

Answer : A
sialotithiasis more common in Submandibular as more viscid secretion and the duct maybe
block by food

Pleomorphic adenomas of the salivary glands are characterized by one of the following
?(4/2017)

A . They occur only in the parotid glands .


B . They grow rapidly .
C . They rarely recur if simply enucleated .
D . They present as rock _ hard masses .
E . They are common benign salivary tumors .

Answer : E
- Pleomorphic adenomas also present with submandibular gland
- grow slowly
- high recurrence with simply enucleation so need total parotiectomy
- Swelling Well-defined, lobulated, freely mobile (not attached to skin, muscles or bones)
variable in consistency (firm or cystic but never hard) elevating lobule of ear with no cervical
LNs enlargement or fascial nerve infiltration

All are true about warthin tumor except ? (1/2017)


A . Benign cystic tumor of the salivary glands .
B . Malignant potential 20%
C . Smoking is a major risk factor
D . Slowly growing painless mass at the angel of the mandible
E . Old age 60_70 years male predominance

4560045770 ‫الدكتور يزن ابو غربيه‬


Page 108
Answer : B
warthin tumor has low rate of malignant transformation < 1%

All of the following are true regarding nick lumps except : (10/2016)
A) Thyroglossal cyst moves upward on protrusion of tongue
B) Branchial cysts typically present in the first decade of life
C) Thyroid masses move upwards on swallowing
D) Cystic hygromas are lymphatic malformation
E) Parotid glands are not normally palpable

Answer : E
parotid may normally palpable

A strong correlation exists between Nasopharyngeal carcinoma and : (10/2016)


A) White race
B) EBV
C) Betel nut chewing
D) Alcohol
E) Plummer Vinson syndrome

Answer : B
nasopharyngeal CA is the most common cancer originating in the nasopharynx, most
commonly in the postero-lateral nasopharynx or pharyngeal recess or 'Fossa of Rosenmüller'

Management of nasopharyngeal carcinoma consist primarily by ? (7/2016)


A . Radiation
B . Chemotherapy
C . Surgical excision
D . Combined chemo_radio
E . Combined chemo_radio followed by surgery

Answer : D
Radiation therapy is the mainstay of treatment, with chemotherapy used in advanced cases.
Concurrent cisplatin, 5-fluorouracil, and radiotherapy have been shown to improve
survival. Sequential chemoradiotherapy with gemcitabine and cisplatin has been shown to
improve survival in locoregionally advanced nasopharyngeal carcinoma. Many pediatric
studies have used neoadjuvant chemotherapy followed by radiation therapy with improvement
in local control or progression-free survival rates over radiotherapy alone

A 65 year old male presents with a persistent firm lateral neck mass that measures
approximately 2.5 cm . The best next step in management is ? (7/2016)
A . Positron emission tomography .
B . CT scan of the head and neck .
C . Fine needle aspiration cytology .

4560045770 ‫الدكتور يزن ابو غربيه‬


Page 109
D . Chest radiography .
E . Excision of the neck mass .

Answer : C
first step in neck mass management to do FNAB or US

All are true about carcinoma of the lip except ? (7/2016)


A . 95% of lip carcinoma in the Lower lip .
B . Sun exposure and smoking are risk factors .
C . More common in elderly white men .
D . Radiation therapy is the treatment of choice .
E . The majority presents as ulcerative lesion

Answer : D
lip carcinoma need surgical excision

All are true about salivary glands neoplasms except ? (7/2016)


A . Mostly arise in the Parotid gland .
B . Warthin tumors are the second most common benign salivary glands tumor and strongly
related to smoking .
C . Mucoepidermoid carcinoma is the most common malignant salivary glands tumor
D . Pleomorphic adenoma does not undergo malignant degeneration .
E . Minor salivary glands tumor are more likely to be malignant than submandibular gland
tumors .

Answer : D
Pleomorphic adenoma is a common benign salivary gland neoplasm characterised by
neoplastic proliferation of parenchymatous glandular cells along with myoepithelial
components, having a malignant potentiality. It is the most common type of salivary gland
tumor and the most common tumor of the parotid gland.
It derives its name from the architectural Pleomorphism (variable appearance) seen by light
microscopy. It is also known as "Mixed tumor, salivary gland type", which refers to its dual
origin from epithelial and myoepithelial elements as opposed to its pleomorphic appearance

4560045770 ‫الدكتور يزن ابو غربيه‬


Page 110

You might also like